ANTR510 W6 Qbank

Ace your homework & exams now with Quizwiz!

D: Orbicularis oris. The orbicularis oris is a muscle of facial expression (mimetic muscle), and these mus- cles insert or originate in the subcutaneous tissue of the skin, which enables them to convey mood via facial expressions as well as alter the form of the facial orifices. All the muscles of facial expression are derived from the mesoderm of the second pharyngeal (hyoid) arch and innervated by the facial nerve (CN VII). Most of these muscles are located within the face; however, a few extend into the scalp (e.g., epicranius) or the neck (platysma). The most important, and therefore most clin- ically relevant, role of the facial muscles is to control and oper- ate the facial orifices, that is, the eyes, nose, mouth, and ears. Facial expression is a secondary byproduct of the ability to finely control the facial orifices. The mouth is controlled by an extensive, interweaving array of facial muscles that influence feeding, respiration, and articulate speech. The orbicularis oris is a broad, very complex muscle that encircles the mouth in a sphincter-like fashion. It interacts with the other orofacial muscles to modify the form and tension of the lips and their margins. By itself, it acts to close the mouth by bringing the lips together tightly. Choice A (Zygomaticus major) is incor- rect. The zygomaticus major is an elongated muscle of facial expression that originates on the lateral aspect of the zygomatic bone and attaches into the angle of the mouth. It contracts to dilate oral fissure, but it also elevates the labial commissures bilaterally to smile (show happiness) or unilaterally to sneer (show disdain). Choice B (Buccinator) is incorrect. The buc- cinator (L: trumpeter) muscle is a wide, thin muscle of facial expression that lies relatively deep in the cheek, coursing from the pterygopalatine raphe and the alveolar ridges of maxil- lary and mandibular molar teeth to insert into the orbicularis oris at the angle of the mouth. The buccinator compresses the cheeks and lips against the teeth and gums, provides resis- tance to keep the teeth from tilting laterally, and prevents patients from looking like a hamster when they chew food. This muscle is important in all phases of feeding. During mas- tication, it assists in positioning food between the occlusal surfaces of the teeth. In suckling, it creates pressure within the oral cavity. This pressure also serves in blowing air, as when playing a wind instrument. Choice C (Levator labii superio- ris) is incorrect. The levator labii superioris is a relatively large muscle of facial expression that runs from the infraorbital margin of the maxilla into the upper lip. Acting in concert with its neighbors, it dilates the oral fissure by elevating and everting the upper lip to show the upper (maxillary) teeth. It also deepens nasolabial sulcus to show sadness. Please note that this muscle covers the infraorbital foramen, lying over the emerging infraorbital nerve and vessels. Choice E (Mentalis) is incorrect. The mentalis is a small, conical muscle of facial expression that originates on the incisive fossa of the mandible and inserts into the skin of the chin and the base of the lower lip. When pouting, this muscle raises, protrudes, and everts the lower lip. This muscle also elevates the skin of the chin to show doubt.

As a result of facial injuries suffered in an automobile accident, a 17-year-old girl is unable to close her lips together tightly. Which of the following muscles is paralyzed? (A) Zygomaticus major (B) Buccinator (C) Levator labii superioris (D) Orbicularis oris (E) Mentalis

B The inferior thyroid veins pass anterior to the trachea on their way to the left brachiocephalic vein.

5-year-old boy fell from a tree and was admitted to the emergency department unconscious. When an emergency tracheostomy was performed, profuse dark venous bleeding suddenly occurred from the midline incision over the trachea. Which of the following vessels was most likely accidentally cut? A Superior thyroid vein B Inferior thyroid vein C Left brachiocephalic vein D Middle thyroid vein

B When in primary position the resting tone (contraction) of the medial rectus muscle is unopposed and results in adduction of the eye.

In performing an H-test on a patient you find that when asking the patient to abduct an eye from an adducted position, the pupil does not move beyond primary position. How will the affected eye in the patient above present when in primary position? A Abducted pupil B Adducted pupil C Laterally rotated pupil D Medially rotated pupil

C: Cricothyroid membrane. In an emergency tracheotomy, the cricothyroid membrane is incised in order to establish a direct airway for the patient. This procedure is also called a cricothyrotomy or cricothyroidotomy, and it is used as a last resort to circumvent upper airway obstructions. The cri- cothyroid membrane is an important component of the conus elasticus, which is composed of the vocal ligaments, median cricothyroid membrane, and lateral cricothyroid membranes. The cricothyroid membrane is the perfect location to perform an emergency tracheotomy because of several nearby palpa- ble landmarks, and it is located below the (true) vocal folds, which serve as the main inspiratory sphincter of the larynx. Do not confuse the emergency tracheotomy with a tracheos- tomy, which is a procedure performed in a hospital setting and involves surgically creating a hole in the cartilaginous rings of the trachea. Choice A (Cricoid cartilage) is incorrect. Due to the thickness of the cricoid cartilage, incising through this car- tilage would be difficult outside the hospital setting. Moreover, an incised cricoid cartilage would need surgical intervention to heal due to its lack of blood supply. Moreover, damaging the cricoid cartilage would be detrimental to the integrity of the larynx and the laryngeal skeleton. Choice B (Thyrohyoid membrane) is incorrect. Though it is easily palpated due to its position above the laryngeal prominence (or the "Adam's apple"), the thyrohyoid membrane is located between verte- bral levels C3 and C4 and may not establish a direct airway. To combat against an upper airway obstruction, the cricothy- roid membrane (between vertebral levels C5 and C6) would be a better option. Damage to the thyrohyoid membrane could also compromise the superior laryngeal artery and the inter- nal branch of the superior laryngeal nerve, which pierce this membrane to enter the larynx. Choice D (Tracheal rings) is incorrect. In a tracheostomy, a hole is created surgically in the cartilaginous rings of the trachea. However, this procedure is usually performed in a hospital setting under sterile conditions. Damage to the thyroid gland and infrahyoid muscles can easily occur if a tracheostomy is not performed correctly. Incising the cricothyroid membrane would be a much easier means of estab- lishing an airway, especially considering the emergency condi- tions surrounding this patient's choking incident. Choice E (Isthmus of the thyroid gland) is incorrect. Cutting through the isthmus of the thyroid gland would not establish an airway for this patient, so this option can be easily eliminated. Due to its location at the seventh cervical vertebra, this glandular tissue is often transected or resected during a tracheostomy, when the tracheal rings are incised. However, a tracheostomy is performed in a hospital setting under sterile conditions.

A 68-year-old man was choking on a piece of steak at a family restaurant. Despite attempts to dislodge the food via abdomi- nal thrusts (or the Heimlich maneuver), his upper airway remained blocked. An emergency medical technician (EMT), eating at the scene, performed an emergency tracheotomy to enable the man to breathe. Which subcutaneous structure was most likely cut during this procedure? (A) Cricoidcartilage (B) Thyrohyoid membrane (C) Cricothyroidmembrane (D) Trachealrings (E) Isthmus of thyroid gland

D: Superior cervical ganglion. The student's reaction is a classic sympathetic "fight, fright, flight" response. All sympathetic fibers destined for the head synapse in the superior cervical ganglion. This ganglion marks the cranial end of the sympathetic chain, and is located near the base of the skull, at about vertebral level C2. Presynaptic neurons ascend through the sympathetic chain to the superior cervical ganglion. There, they synapse with postsynaptic neuron cell bodies that distribute their axonal processes throughout the head. These sympathetic fibers head control vasomotion, pilo- motion, and sudomotion. Choice A (Ciliary ganglion) is incor- rect. The ciliary ganglion is a parasympathetic ganglion, located in the orbit. The fibers that synapse in this ganglion supply the ciliary and sphincter pupillae muscles in the eye. It has no input into this sympathetic response. Choice B (Trigeminal ganglion) is incorrect. The trigeminal (semilunar) ganglion is a sensory ganglion. It houses the cell bodies of the pseudounipolar neu- rons carrying general sensory information from the trigeminal nerve (CN V). It has no input into this sympathetic response. Choice C (Inferior [nodose] vagal ganglion) is incorrect. The vagus nerve possesses two ganglia, superior and inferior, which are sensory ganglia. The superior vagal ganglion houses cell bodies of general sensory neurons. The inferior vagal ganglion houses the cell bodies of visceral afferent neurons. It has no input into this sympathetic response. Choice E (Pterygopala- tine ganglion) is incorrect. The pterygopalatine ganglion is a parasympathetic ganglion, located within the pterygopalatine fossa. It sends fibers to the lacrimal gland and glands within the nasal cavity. It has no input into this sympathetic response.

A male 1st-year medical student mistakenly enters the women's locker room and finds a group of his female colleagues changing their clothes. He is shocked and embarrassed by his mistake and immediately runs away with his heart pounding. Given his agitated state, what ganglion, housing neuron cell bodies, is experiencing an extremely high rate of activity? (A) Ciliaryganglion (B) Trigeminal ganglion (C) Inferior (nodose) vagal ganglion (D) Superior cervical ganglion (E) Pterygopalatine ganglion

C: Inability to close the eyelids. The stylomas- toid foramen is located in a well-protected position between the styloid and mastoid processes, at the base of the skull. It is the terminal opening of the facial canal, transmitting the main branch of the facial nerve out of the petrous temporal bone to the exterior base of the skull. From there, the facial nerve sends sensory branches to the external ear, motor branches to small muscles in the upper neck (stylohyoid and posterior belly of the digastric), and terminal motor branches to the facial muscles (muscles of facial expression). However, the mastoid process is not developed at birth, leaving the stylomastoid foramen and the emerging facial nerve exposed. Undue pres- sure applied to the mastoid area (as from misapplied obstet- ric forceps) may entrap and damage the facial nerve. In this case, crush injury to the facial nerve would produce unilateral paralysis of the facial muscles, including the orbicularis oculi. This sphincter-like muscle surrounds the orbit, acting to close the eyelids. Choice A (Reduced blood flow to the inner ear) is incorrect. Vascular supply to the inner ear is provided by small labyrinthine vessels that accompany the facial and ves- tibulocochlear nerves through the internal acoustic meatus. Supply to the external ear is mainly from branches of the pos- terior auricular and superficial temporal arteries. The posterior auricular arterial branching is susceptible to damage in this scenario. Supply to the middle ear is mainly from branches of the maxillary artery. These vessels are unlikely to be affected by compression around the incipient mastoid process. Choice B (No sensation in the external acoustic meatus) is incorrect. The external ear receives afferent innervation from a plethora of cranial nerves, including trigeminal (CN V), facial (CN VII), vagus (CN X), and possibly even the glossopharyngeal (CN IX). The trigeminal and vagus are the major nerves, supplying most of the sensory fibers to this region, including the exter- nal acoustic meatus. The facial nerve provides relatively small contributions to the sensation of the external acoustic meatus because its sensory innervation is concentrated on the auricle. Choice D (Lack of taste on the body of the tongue) is incor- rect. The facial nerve provides taste fibers to the body (anterior two thirds) of the tongue and parasympathetic fibers to the oral floor via its chorda tympani branch. However, the chorda tympani nerve leaves the facial nerve within the facial canal, traverses the tympanic (middle ear) cavity, and exits the base of the skull near the temporomandibular joint (TMJ). Therefore, this forceps injury would not affect the taste or parasympathetic functions governed by the facial nerve. Choice E (Inability to tense the eardrum) is incorrect. The tensor tympani muscle attaches to the handle of the malleus, acting to pull this ear ossicle medially and tense the tympanic membrane (eardrum). This muscle is innervated by the mandibular division of the trigeminal nerve (CN V3). The stapedius muscle attaches to the stapes, acting to tighten the oval window. This muscle is controlled by the facial nerve, via a branch off the facial canal, proximal to the stylomastoid foramen.

During a difficult childbirth, a physician uses obstetric forceps to grip the infant's head as an aid to extracting her from the birth canal. However, the forceps are misapplied at the right stylomastoid foramen and crush its contents at the opening of the foramen. Which of the following ipsilateral deficits is the baby most likely to suffer? (A) Reduced blood flow to the inner ear (B) No sensation in the external acoustic meatus (C) Inability to close the eyelids (D) Lack of taste on the body of the tongue (E) Inability to tense the eardrum

B: Paresthesia of the lower lip. The inferior alveolar nerve, a branch of the mandibular division of the trigeminal nerve (CN V3), leaves the ramus of the mandible after traveling through the extent of that bone, conveying afferent fibers from the mandibular teeth. Its mental nerve branch enters the mandible through the mental foramen after supplying sensory (cutaneous) innervation to the skin overly- ing the mandible, including the lower lip. Due to its location within the mandible and close relation to the dental arcade, the inferior alveolar nerve may be damaged during dental extrac- tions, especially more complicated extractions of impacted third molars ("wisdom teeth"). Thus, paresthesia in the lower lip may result from such procedures. Choice A (Inability to compress the cheek) is incorrect. Compression of the cheek is an important action in all aspects of feeding and in other oral functions as well, and it is produced by the contraction of the buccinator muscle, one of the facial muscles. Being a muscle of facial expression, the buccinator is controlled by the facial nerve (CN VII), through its terminal facial branches. Choice C (Weakness in closing the jaw) is incorrect. Chew- ing is the result of the complex interactions of the muscles of mastication, which are controlled by the motor branches of the mandibular division of the trigeminal nerve (CN V3). Of these four mastication muscles, the masseter, temporalis, and medial pterygoid muscles act in closing the jaw. The lateral pterygoid and anterior belly of the digastric muscles act in the opening of the oral fissure by depressing the mandible. The inferior alveolar nerve gives rise to the mylohyoid nerve before it enters the mandible via the mandibular foramen. The mylohyoid nerve supplies the anterior belly of the digas- tric and mylohyoid muscles. Therefore, while damage to the proximal part of the inferior alveolar nerve may influence jaw opening by affecting the anterior digastric muscle, damage to its distal (intramandibular) part does not affect any muscles of mastication, sparing both opening and closing of the mandible. Choice D (Decreased salivary flow) is incorrect. Secretomotor control of the submandibular and sublingual salivary glands is governed by parasympathetic neurons that originate in the facial nerve through its chorda tympani branch and ultimately reach the oral floor via the lingual nerve, a branch of the mandibular division of the trigeminal nerve (CN V3). Control of the parotid salivary gland is conveyed through parasym- pathetic fibers that originate in the glossopharyngeal nerve, travel through its tympanic and lesser petrosal branches, and ultimately reach the parotid through the auriculotemporal nerve (another branch of CN V3). Choice E (Reduced taste in the body of the tongue) is incorrect. Taste fibers in the body (anterior two thirds) of the tongue are provided by the facial nerve via its chorda tympani branch. General sensory fibers to the same area are supplied by the lingual nerve, a branch of the mandibular division of the trigeminal nerve (CN V3). The chorda tympani nerve unites with the lingual nerve within the infratemporal fossa. Thus, whereas the taste and general sensory components to the body of the tongue arise from separate cranial nerve pathways, they share a final common pathway to their target region. Remember that the parasympa- thetic supply to the oral floor (submandibular and sublingual salivary glands; oral floor mucus glands) also shares most of the chorda tympani/lingual nerve pathway. However, the auto- nomic fibers leave the lingual nerve in the oral floor to synapse in the submandibular ganglion.

During extraction of her impacted wisdom teeth, a 22-year-old woman suffers damage to her right inferior alveolar nerve. Which of the following conditions is most likely to result? (A) Inability to compress the cheek (B) Paresthesia of the lower lip (C) Weakness in closing the jaw (D) Decreased salivary flow (E) Reduced taste in the body of the tongue

C The auriculotemporal branch of the mandibular division of the trigeminal nerve (CN V3) innervates the skin anterior to the ear and superiorly in the adjacent temporal region.

In performing a neurological assessment of a 73-year-old female patient the physician lightly touches the temporal region immediately superior to the ear with a piece of cotton. Which of the following sensory domains is being tested in this patient? A V1 dermatome B V2 dermatome C V3 dermatome D C2 dermatome E C3 dermatome

A Abduction of the eye occurs through the combination of increased activity of the lateral rectus muscle, and decreased activity (inhibition) of the medial rectus muscle. In the patient described here, abducting the eye beyond primary position is not possible due to injury to the abducens nerve and absence of activation of the lateral rectus muscle. The affected eye successfully abducted to primary position due to inhibition of medial rectus muscle contraction, which is controlled by the intact oculomotor nerve.

In performing an H-test on a patient you find that when asking the patient to abduct an eye from an adducted position, the pupil does not move beyond primary position. Which of the following cranial nerves may be injured in this patient? A Abducens B Oculomotor C Ophthalmic D Optic E Trochlear

A. facial nerve. Along with the muscles of facial expression, posterior belly of the digastric, and sty- lohyoid muscles, the facial nerve also innervates the stapedius muscle, which contracts to pull the stapes away from the oval window of the cochlea. This action lowers the amplitude of sounds waves and decreases the transmission of vibrations to the cochlea. Paralysis of the stapedius results in hyperacusis due to heightened reaction of the stapes to sound vibration. Therefore, a person would be more sensitive to loud sounds, particular low-frequency sounds, like the bass emitted from a subwoofer. The stapedius muscle contracts involuntarily, along the tensor tympani muscle innervated by CN V3, in response to high intensity sound waves, forming the acous- tic reflex. Choice B (Vestibulocochlear nerve) is incorrect. Because of the normal results from the audiometer hear- ing examination, the vestibulocochlear nerve (CN VIII) has not been damaged. However, the stapedius muscle, which decreases the impact of the footplate of the stapes on the oval window of the cochlea, has been injured. Given this patient's loss of the acoustic reflex, extended exposure to loud sounds could damage the cochlea in the future. Choice C (Glossopharyngeal nerve) is incorrect. The glossopharyngeal nerve (CN IX) supplies only one muscle, the stylopharyn- geus, and it is not involved with the acoustic reflex or sound transduction. So, the hyperacusis seen in this patient is not due to CN IX. Choice D (Vagus nerve) is incorrect. The vagus nerve (CN X) supplies all the muscles of the larynx as well as most of the muscles of soft palate (exception = tensor veli palatini of CN V3) and pharynx (exception = stylopharyngeus of CN IX). Despite its extensive motor innervation, the vagus nerve does not play a role in the acoustic reflex or sound conduction. Choice E (Hypoglossal nerve) is incorrect. The hypoglossal nerve (CN XII) innervates the intrinsic and (most of the) extrinsic muscles of tongue (exception = palatoglossus muscle of CN X). Losing innervation to the muscles of the tongue would cause ipsilateral tongue deviation, but would not cause hyperacusis.

Most senior citizens dislike the bass audio frequencies augmented by a large subwoofer in a teenager's car. But, one 68-year-old man came to his doctor complaining of hyperacusis, or heightened sensitivity to these loud, low-frequency sounds. The results from an audiometer hearing test reveal normal hearing for a man of his age. Given the presentation, what cranial nerve is most likely involved with his hyperacusis? (A) Facial nerve (B) Vestibulocochlear nerve (C) Glossopharyngeal nerve (D) Vagus nerve (E) Hypoglossal nerve

C the cells within the pterygopalatine ganglion innvervate the lacrimal gland and the mucus-producing glands of the nose and palate. the greater petrosal br. of the facial n. provides the preganglionic input to the cells in the pterygopalatine ganglion.

The cell bodies of the postganglionic parasympathetic neurons supplying the lacrimal gland are located in which of the following ganglia? A. Ciliary B. Otic C. Pterygopalatine D. Superior cervical E. Trigeminal

D: Auricle of the external ear. Unilateral facial paralysis can result from brain tumor, stroke, or a virus, such as herpes zoster. When no specific cause can be found (idio- pathic), the condition is called facial (Bell or CN VII) palsy. When differentiating between unilateral facial paralysis and herpes zoster infection of CN VII, the physician must imme- diately look for herpetic lesions in the skin supplied by the cutaneous innervation of the facial nerve. CN VII supplies cutaneous innervation to only a small portion of the auricle of the external ear, via the posterior division of the main trunk of the facial nerve. The geniculate ganglion of the facial nerve would be affected by the viral infection known as herpes zoster, or varicella zoster virus. This virus causes chickenpox, and after exposure to chickenpox, this virus can reside latent in ganglia of an individual for years. If this individual becomes immunocompromised, the skin (or dermatomes supplied by the infected ganglia) can develop shingles, a painful skin rash, which blisters, breaks open, crusts over, and then disappears. For CN VII, these herpetic lesions are found in the auricle of the external ear, which is the only cutaneous distribution site for the facial nerve. In Ramsay Hunt syndrome, a patient presents with Zoster eruptions in the auricle or external acoustic meatus of the ear and unilateral facial paralysis. This syndrome is often associated with other CN VII and CN VIII symptoms, including loss of taste to the anterior two thirds of the tongue (CN VII), tinnitus (CN VIII), vertigo (CN VIII), or hearing loss (CN VIII). Choice A (Mental region of the mandible) is incorrect. The mental region of the mandible is supplied by the mental nerve, a cutaneous nerve off the man- dibular division of the trigeminal nerve (CN V3). If the herpes zoster virus infected the trigeminal (or semilunar) ganglion, herpetic lesions could be found along the sensory distribution pattern of CN V3; however, in this patient, the facial nerve (CN VII) was affected due to unilateral paralysis of the facial muscles. Choice B (Temporal and parietal region of the scalp) is incorrect. The temporal and parietal regions of the scalp are supplied by the auriculotemporal nerve, a cutaneous nerve off the mandibular division of the trigeminal nerve (CN V3). If the herpes zoster virus infected the trigeminal (or semilunar) ganglion, herpetic lesions could be found along the sensory distribution pattern of CN V3; however, in this patient, the facial nerve (CN VII) was affected due to unilateral paralysis of the facial muscles. Choice C (Upper lip and cheek of the face) is incorrect. The upper lip and cheek of the face are sup- plied by the infraorbital nerve, a cutaneous nerve off the max- illary division of the trigeminal nerve (CN V2). If the herpes zoster virus infected the trigeminal (or semilunar) ganglion, herpetic lesions could be found along the sensory distribution pattern of CN V2; however, in this patient, the facial nerve (CN VII) was affected due to unilateral paralysis of the facial muscles. Choice E (Bridge and tip of the external nose) is incorrect. The bridge and tip of the external nose are supplied by the infratrochlear and external nasal nerves, respectively. These two nerves are cutaneous branches off the ophthalmic division of the trigeminal nerve (CN V1). If the herpes zoster virus infected the trigeminal (or semilunar) ganglion, herpetic lesions could be found along the sensory distribution pattern of CN V1; however, in this patient, the facial nerve (CN VII) was affected due to unilateral paralysis of the facial muscles.

To differentiate between idiopathic unilateral paralysis of the muscles of facial expression (Bell palsy) and a herpes zoster infection of CN VII, the physician must look for small her- petic lesions (vesicles or blisters). Where are these skin lesions located in a herpes zoster infection involving the facial nerve? (A) Mental region of the mandible (B) Temporal and parietal region of the scalp (C) Upper lip and cheek of the face (D) Auricle of the external ear (E) Bridge and tip of the external nose

B the bridge and dorsum of the nose are part of the opthalmic dermatome of the face; the lateral aspects of the external nose are part of the maxillary dermatome

Two weeks following rhinoplasty (cosmetic surgery on the nose) a 24-year-old female states that she has numbness and tingling from the bridge to the tip of her nose. Branches of which of the following nerves were most likely injured in the rhinoplasty? A. Olfactory B. Ophthalmic C. Maxillary D. Mandibular E. Facial

A the lingual br. of the mandibular n. (CNV-3) provides general SENSATION (not taste) to the mucosa of the anterior 2/3 of the tongue

Following radical resection of a primary tongue tumor, a 72-year-old patient has lost general sensation to the anterior portion of his tongue. This is probably due to injury to branches of which of the following nerves? A. Mandibular B. Chorda Tympani C. Glossopharyngeal D. Hypoglossal E. Vagus

F The abducens nerve (CN VI) travels through the cavernous sinus on the lateral aspect of the internal carotid artery. The abducens nerve is the only nerve located within the cavernous sinus.

A 54-year-old man is admitted to the hospital due to severe headaches. A CT examination reveals an internal carotid artery aneurysm inside the cavernous sinus. Which of the following nerves is most likely to be affected by this aneurysm? A Optic (CN II) B Oculomotor (CN III) C Trochlear (CN IV) D Ophthalmic (CN V1) E Maxillary (CN V2) F Abducens (CN VI)

A. The loose areolar connective tissue layer is known as the "danger zone" because hematoma can spread easily from this layer into the skull by means of emissary veins that pass into and through the bones of the skull. None of the other scalp layers listed is referred to as the "danger zone." GAS 874; GA 442

114 A 55-year-old man is admitted to the emergency department after slipping on wet pavement and falling. Physical examination reveals that the patient has a hematoma that formed in the danger zone of the scalp, spreading to the area of the eyelids. Which of the following layers is regarded as the "danger zone"? ⃣ A. Loose, areolar layer ⃣ B. Skin ⃣ C. Galea aponeurotica ⃣ D. Pericranium ⃣ E. Subcutaneous layer

B the "danger zone" of the scalp consists of the loose alreolar layer located between the periosteum of the cranium and the occipitofrontalis muscle (and its aponeurosis)

A 55-year-old male is admitted to the hospital after an injury sustained at work in a factory. He presented with severe scalp lacerations, which were sutured. After three days the wound is inflamed, swollen, and painful. Between which tissue layers is the infection most likely located? A. The periosteum and bone B. The aponeurosis and the periosteum C. The dense connective tissue and the aponeurosis D. The dense connective tissue and the skin

A. The superior cerebellar artery arises near the termination of the basilar artery, passes immedi- ately below the oculomotor nerve, and eventually winds around the cerebral peduncle, close to the trochlear nerve, as it continues on toward the upper surface of the cerebellum where it will divide into branches that anastomose with the inferior cerebellar arteries. The trochlear nerve passes between the pos- terior cerebral artery and the superior cerebellar ar- tery, and therefore a hematoma of the superior cere- bellar artery can easily injure the trochlear nerve, which runs alongside the internal carotid artery and then enters the orbit through the superior orbital fis- sure. The facial and vestibulocochlear nerves both enter the skull via the internal acoustic meatus (or internal auditory meatus) in the temporal bone and do not have an intimate relationship with the superior cerebellar artery. The glossopharyngeal nerve passes through the jugular foramen, and as it exits from the skull it passes forward between the internal jugular vein and internal carotid artery. GAS 837, 838; GA 451-452, 454

131 A 64-year-old man is admitted to the hospital in an unconscious condition. A CT scan examination reveals that the patient has suffered a cerebral vascular accident (stroke), with a small hematoma produced by the superior cerebellar artery. Which of the following nerves will most likely be affected by the hematoma? ⃣ A. Trochlear nerve ⃣ B. Abducens nerve ⃣ C. Facial nerve ⃣ D. Vestibulocochlear nerve ⃣ E. Glossopharyngeal nerve

E. Within the cavernous sinus the abducens nerve is in intimate contact with the internal carotid artery. Therefore, an aneurysm of the internal carotid artery could quickly cause tension or compression on the abducens nerve. This would result in ipsilateral paralysis of abduction of the pupil. Inability to gaze downward and medially would be due to the trochlear nerve, which is not in the cavernous sinus. Complete ptosis would be a result of a complete lesion in the oculomotor nerve, which is not apparent here. Bilateral loss of accommodation and loss of pupillary reflex would be the result of bilateral loss of the oculomotor nerve, which is not likely in this situation. Finally, ip- silateral loss of the consensual corneal reflex is a result of loss of both the ophthalmic division of the trigemi- nal nerve and the facial nerve, supplying the afferent and efferent limbs of the reflex, respectively. GAS 844; GA 445, 450, 452

138 A 65-year-old woman is admitted to the hospital with signs of cavernous sinus thrombosis. Radiographic examination reveals an aneurysm of the internal carotid artery within the cavernous sinus. During physical examination what sign would one fi rst expect to see if nerve compression has occurred within the cavernous sinus? ⃣ A. Inability to gaze downward and medially on the affected side ⃣ B. Complete ptosis of the superior palpebra ⃣ C. Bilateral loss of accommodation and loss of direct pupillary refl ex ⃣ D. Ipsilateral loss of the consensual corneal refl ex ⃣ E. Ipsilateral paralysis of abduction of the pupil

A. The anterior division of the facial nerve passes through the parotid gland and is therefore at risk during surgery of the parotid gland. Since this patient's symptoms involved paralysis of the mus- cles of the lower lip, the branch of the facial nerve that supplies these muscles, the marginal mandibular branch, is the one that has suffered the iatro- genic injury.GAS 865; GA 456, 458-459, 480

141 A 55-year-old woman has undergone facial surgery for the excision of a malignant parotid tumor. A week postoperatively, marked weakness is seen in the musculature of the patient's lower lip. Which of the following nerves was most likely injured during the parotidectomy ? ⃣ A. Marginal mandibular branch of facial ⃣ B. Zygomatic branch of facial ⃣ C. Mandibular division of the trigeminal nerve ⃣ D. Buccal branch of facial ⃣ E. Buccal nerve

E. The jugular foramen is the route of exit for three nerves (glossopharyngeal, vagus, and accessory nerves) and one vein (internal jugular) from the cranial cavity. The glossopharyngeal nerve provides the sensory input for the gag refl ex, whereas the vagus nerve provides the motor output. Nerve compression within this foramen would lead to a loss of both systems and thus no gag refl ex. Tongue movements are supplied by the hypoglossal nerve, which exits the skull through the hypoglossal canal. The facial nerve innervates the muscles of the face and would not be affected by this injury. Loss of sensation from the face and scalp would be present only if there was involvement of the trigeminal nerve. Loss of hearing would

25 A 70-year-old man is admitted to the hospital with chronic headache and enlarged lymph nodes. A CT scan shows a tumor at the jugular foramen. Which of the following would be the most likely neurologic defi cit? ⃣ A. Loss of tongue movements ⃣ B. Loss of facial expression ⃣ C. Loss of sensation from the face and the scalp ⃣ D. Loss of hearing ⃣ E. Loss of gag reflex

A. An injury to the left vagus nerve would cause the uvula to become deviated to the right. This is because of the innervation of the musculus uvulae muscle that makes up the core of the uvula. If only one side is effectively innervated, contraction of the active muscle will deviate the uvula to the contralateral side of the injury (ipsilateral side of the uninjured vagus nerve). In addition, the intact levator veli palatini will pull the uvula to the intact side. The right and left hypoglossal nerves innervate the tongue muscles and would not affect the uvula. The glossopharyngeal nerve supplies sensory innervation to the oropharynx and nasopharynx, but not motor innervation to these areas.

29 A 36-year-old woman is admitted to the hospital with severe head injuries after a car crash. During neurologic examination her uvula is deviated to the right. Which nerve is most likely affected to result in this deviation? HEAD AND NECK 215 ⃣ A. Left vagus ⃣ B. Right vagus ⃣ C. Right hypoglossal ⃣ D. Left glossopharyngeal ⃣ E. Right glossopharyngeal

The superior ophthalmic vein drains directly into the cavernous sinus. The danger area of the face is located in the triangular region from the lateral angle of the eye to the middle of the upper lip, near the nose, and is drained by the facial vein. The facial vein communicates directly with the cavernous sinus through the superior ophthalmic vein. The pterygoid venous plexus communicates with the cavernous sinus through the inferior ophthalmic vein, but it is not directly connected to the cavernous sinus. The basilar venous plexus connects the inferior petrosal sinuses and communicates with the internal vertebral venous plexus. The parietal emissary veins and frontal venous plexus do not communicate directly with the cavernous sinus.

42 A 35-year-old woman is hospitalized due to cavernous sinus thrombosis resulting from an infection on the face. Which of the following is the most direct route for spread of infection from the face to the cavernous sinus? ⃣ A. Pterygoid venous plexus ⃣ B. Superior ophthalmic vein ⃣ C. Frontal venous plexus ⃣ D. Basilar venous plexus ⃣ E. Parietal emissary vein

C. The external surface of the tympanic mem- brane is innervated primarily by the auriculotemporal nerve, a branch of the mandibular division of the tri- geminal nerve. Damage to this nerve would addition- ally result in painful movements of the TMJ because this joint receives innervation from the same nerve. Taste in the anterior two thirds of the tongue is sup- plied by the facial nerve and would be unaffected in this injury. The sensory innervation of the nasal cav- ity is supplied by the ophthalmic and maxillary divi- sions of the trigeminal nerve and would be unaffected by injury to the tympanic membrane. Sensory inner- vation to the larynx is provided by the vagus nerve, whereas the pharynx receives sensory fibers from the glossopharyngeal and vagus nerves. The palate is supplied by the maxillary divisions of the trigeminal nerve and would be unaffected by this injury. GAS 904; GA 455, 483-485

69 A 3-year-old girl ruptured her eardrum when she inserted a pencil into her external ear canal. She was urgently admitted to the emergency department. Physical examination revealed pain in her ear and a few drops of blood in the external auditory meatus. There was the concern that there might possibly have been an injury to the nerve that principally innervates the external surface of the tympanic membrane. Which of the following tests is most likely to be performed during physical examination to check for injury to this nerve? ⃣ A. Check the taste in the anterior two thirds of the tongue. ⃣ B. Check the sensation to the pharynx and palate. ⃣ C. Check if there is paraesthesia at the TMJ. ⃣ D. Check for sensation in the larynx. ⃣ E. Check for sensation in the nasal cavity.

C. Parasympathetic innervation of the parotid gland is provided by axons carried by the glossopha- ryngeal nerve that emerge from the tympanic plexus of the middle ear as the lesser petrosal nerve. These preganglionic parasympathetic fibers terminate by synapses in the otic ganglion, which supplies the se- cretory parasympathetic innervation to the parotid gland. Glandular secretions of the nasal cavity, soft palate, and lacrimal gland all receive parasympathetic innervation from the fibers of the greater petrosal nerve and would remain intact following a tympanic plexus lesion. Axons for secretory innervation to the sublingual and submandibular glands are carried by the facial nerve, then course through the chorda tym- pani, before synapsing in the submandibular gan- glion, with postganglionic fibers eventually reaching the glands via the lingual nerve. GAS 863-865; GA 456, 458-459, 480

72 A 12-year-old girl is admitted to the emergency department with a middle ear infection. Physical examination reveals a long history of chronic middle ear infections that have produced a lesion in the tympanic plexus in the middle ear cavity. Since the preganglionic parasympathetic fi bers that pass through the plexus have been lost, which of the following conditions will be detectable during physical examination? ⃣ A. Diminished mucus in the nasal cavity ⃣ B. Diminished mucus on the soft palate ⃣ C. Diminished saliva production by the parotid gland ⃣ D. Diminished saliva production by the submandibular and sublingual glands ⃣ E. Diminished tear production by the lacrimal gland

E. The superior oblique muscle turns the pupil downward from the adducted position. Inability to perform this motion, in conjunction with diplopia when walking down stairs, indicates damage to the trochlear nerve. The abducens innervates the lateral rectus, resulting in abduction of the eye. The oculo- motor nerve supplies the superior, inferior, and me- dial rectus as well as the inferior oblique. Overall, innervation from the oculomotor nerve results in up- ward and downward movements of the eye. Damage to this nerve would not induce diplopia when an af- fected individual walks down stairs. In addition, in- ability to gaze downward in the adducted position does not indicate oculomotor nerve damage. In this position the oculomotor nerve would be responsible for upward movement. The nasociliary nerve is a sen- sory nerve originating from the ophthalmic branch of the trigeminal nerve. GAS 888-891; GA 464, 466

76 A 43-year-old male is admitted to the hospital complaining of diplopia (double vision) when walking down stairs. During physical examination of the extraocular muscles the patient experiences diplopia, and when he is asked to turn his right eye inward toward his nose and look down, he is able to look inward but not down. Which nerve is most likely involved? ⃣ A. Abducens ⃣ B. Nasociliary ⃣ C. Oculomotor, inferior division ⃣ D. Oculomotor, superior division ⃣ E. Trochlear

B. The posterior chamber receives ciliary body secretions first. The ciliary body produces aqueous humor and is located in the posterior chamber. In- creased production of fluid from this site would cause an increase in intraocular pressure if drainage is inadequate. The iridoscleral angle of the anterior chamber is the location of drainage of the aqueous humor; therefore, a blockage of drainage in this loca- tion can cause increased intraocular pressure. The pupil is the connection between the anterior and posterior chamber; a collection of fluid does not oc- cur here, for this is simply an aperture to allow light onto the retina. The vitreous body is not directly connected to the production of aqueous humor. The lacrimal sac is the upper dilated end of the nasolac- rimal duct and opens up into the inferior meatus of the nasal cavity. The nasolacrimal duct has nothing to do with increased intraocular pressure. GAS 898-899; GA 468

85 A 62-year-old man is admitted to the hospital with blurred vision. Physical examination reveals a long history of gradual loss of his visual fi eld. The intraocular pressure is high, and a diagnosis of glaucoma is made. Which of the following spaces fi rst receives the aqueous humor secreted by the epithelium of the ciliary body? A. Anterior chamber ⃣ B. Posterior chamber C. Pupil ⃣ D. Vitreous ⃣ E. Lacrimal sac

A. The motor portion of the facial nerve (CNVII) innervates the orbicularis occuli muscle. The sphincter muscle of the eyelids is responsible for closing the eyelids- an action that sweeps lacrimal fluids cross the cornea from their origin at the superolateral aspect of the orbit to the lacrimal sac in the inferomedial aspect of the orbit.

A 33-year-old woman develops Bell palsy. She must be cautious because this can result in corneal inflammation and subsequent ulceration. This symptom results from which of the following conditions? A. Absence of the corneal blink reflex B. Absence of sweating on the face C. Inability to constrict the pupil D. Sensory loss of the cornea and conjunctiva

A: Foramen rotundum. The foramen rotundum is an opening in the greater wing of the sphenoid bone that enables the maxillary (second) division of the trigeminal nerve (CN V2) to pass into the middle cranial fossa. CN V2 supplies sensory (cutaneous) innervation to the skin to the lower eye- lid, cheek, upper lip, upper dentition and gingivae, and lateral aspects of the nose. Due to the sensory deficits within this patient, this nerve is damaged along its route, and the foramen rotundum is the most likely location. Choice B (Foramen spinosum) is incorrect. The foramen spinosum conveys the middle meningeal artery and the meningeal branch (nervus spinosum) of the mandibular division of the trigeminal nerve (CN V3). This artery and nerve supply the dura mater in the cranial cavity. Thus, this site has no relation to the deficits in this patient. Choice C (Superior orbital fissure) is incorrect. The superior orbital fissure is a large opening that conveys the oculomotor (CN III), trochlear (CN IV), ophthalmic division of the trigeminal (CN V1), and abducent (CN VI) nerves as well as the superior ophthalmic veins into the orbit. None of these structures have a functional role in the deficits in question. Choice D (Foramen ovale) is incorrect. The foramen ovale (or oval foramen) is an opening in the greater wing of the sphenoid bone that enables the mandibular (third) division of the trigeminal nerve (CN V3) and a small meningeal artery to pass into the middle cranial fossa. CN V3 supplies cutaneous (general) sensation to the lower lip, chin, cheek, and anterior auricular and posterior temporal regions. This sensory inner- vation is supplied via three cutaneous nerves: mental, buccal, and auriculotemporal. The mandibular division of the trigem- inal nerve also supplies innervation to the mandibular teeth and gingivae via the inferior alveolar nerve. Due to the listed sensory deficits in this patient, this nerve would not have been damaged along its route. Choice E (Optic canal) is incorrect. The optic canal carries the optic nerve (CN II) and ophthalmic artery into the orbit. Again, these structures have no bearing on the deficits in this patient.

A 13-year-old girl visits her pediatrician complaining of loss of sensitivity in her lower eyelid, the skin below her right eye, and in her upper lip and teeth. Which of the following sites is the most likely location for the nerve lesion responsible for these signs and symptoms? (A) Foramen rotundum (B) Foramen spinosum (C) Superior orbital fissure (D) Foramen ovale (E) Optic canal

A: Ophthalmic. Both the superior and inferior ophthalmic veins provide a pathway for the spread of infec- tion from the anterior tip of the nose to the cavernous sinus as these veins connect the facial vein to the cavernous sinus. Because the facial vein does not contain valves, infection would spread in the direction of least physical resistance. An infection arising on the tip of the nose may spread to the cavernous sinus causing thrombophlebitis. The "danger tri- angle of the face" stretches from the labial commissures bilat- erally to the bridge of the nose. Any lacerations within this triangle, such as the acne pustules frequently seen in maturing adolescents, can cause an infection in the facial vein, which may spread to the cavernous sinus. Remember that veins of the face and the scalp do NOT contain valves, which is clini- cally relevant due to the potential spread of infection. The symptoms seen in this patient, headache, fever, and perior- bital edema were due to the spread of the infection through the ophthalmic veins to the cavernous sinus. Once the infec- tion resided in the cavernous sinus, it could affect the cranial nerves, oculomotor nerve (CN III), trochlear nerve (CN IV), and abducent nerve (CN VI), which control the extraocular muscles, leading to the diplopia, or double vision, seen in this patient. Choice B (Superior cerebral) is incorrect. The supe- rior cerebral vein drains blood from the superolateral aspects of the cerebrum into the superior sagittal sinus. Though these veins are clinically significant because they are often disrupted from a traumatic blow to the front of head resulting in a sub- dural cerebral hemorrhage, they do not communicate with the cavernous sinus and would not be involved with spread of infection from the tip of the nose. Choice C (Great cere- bral [of Galen]) is incorrect. The great cerebral vein (of Galen) merges with the inferior sagittal dural venous sinus to form the straight dural venous sinus. It is a single, midline vein that drains deep structures of the brain. The great cerebral vein does not communicate with the cavernous sinus and would not drain blood from the tip of the nose. Choice D (Maxillary) is incorrect. The maxillary vein is formed by the drainage of the pterygoid plexus of veins, located between the temporalis and lateral pterygoid muscles in the infratemporal region, and drains into the retromandibular vein. The maxillary vein does not directly communicate with the cavernous sinus; however, it can communicate with the ophthalmic veins through the pterygoid plexus via the inferior orbital fissure. However, the maxillary vein is located some distance away from the "danger triangle of the face" and is unlikely to be involved. Choice E (Supraorbital) is incorrect. The supraorbital vein drains the anterosuperior aspects of the scalp, but it is not involved with venous drainage of the tip of the nose, where the infection originated in this patient. Though this vein does drain into the ophthalmic veins, it is not the correct answer due to the loca- tion of the lesion on the tip of the nose.

A 15-year-old young man developed cavernous sinus thrombophlebitis after a 1-week history of a single acne-like lesion at the anterior tip of his nose. He presented with a headache, periorbital edema, diplopia, and a fever (103°F or 39.4°C). What vein is the most likely route for the spread of this infection to the cavernous sinus? (A) Ophthalmic (B) Superior cerebral (C) Great cerebral (of Galen) (D) Maxillary (E) Supraorbital

D in the H-test, evaluation of the adducted -eye tests for function of the inferior oblique muscle; depression of the adducted eye tests for function of the SUPERIOR OBLIQUE MUSCLE. the superior oblique muscle is innervated by the TROCHLEAR N. (CNIV)

A 19-year-old student presented with a complaint of double vision whenever he attempted to read. When gazing toward the right, the patient could depress the abducted right eye but could not depress the adducted left eye. He could depress the abducted left eye when looking to the left. All other eye movements were normal. Which nerve has been injured? A. Left abducens B. Right abducens C. Left oculomotor D. Left trochlear E. Right trochlear

D In the H-test, adduction of the eye aligns the superior and inferior oblique muscles so as to maximize their ability to produce rotation along the transverse axis, while simultaneously minimizing the ability of the superior and inferior rectus muscles to act on this axis (See University of Toronto H-Test Simulator; https://www.bmc.med.utoronto.ca/anatomia/intro.swf). With the affected eye adducted, depression of the eye occurs through contraction of the superior oblique muscle; elevation of the eye is achieved by contraction of the inferior oblique muscle. An inability to elevate the adducted eye indicates an injury to the inferior oblique muscle.

A 22-year-old male presents at the emergency department following a late night bar brawl in which he was hit squarely in the right eye by a fist of one of the participants. The patient complains of eye pain and double vision, and his eye and surrounding tissues are swollen and bruised. Suspecting a blow-out fracture may have trapped one of the extra-ocular muscles, you manage to successfully complete an H-test on the affected eye. Results indicate that the patient cannot elevate the eye when adducted. Which of the following muscles is likely trapped by the bone fragments of the blow-out fracture? A Superior rectus B Inferior rectus Superior oblique Inferior oblique C Superior oblique D Inferior oblique

D: Masseter. Damage to the facial nerve would lead to loss of innervation to the muscles of facial expres- sion, and the masseter muscle, a muscle of mastication, is the only listed muscle that will continue to function in a patient diagnosed with facial nerve (CN VII or Bell) palsy. The mandib- ular (third) division of the trigeminal nerve (CN V3) supplies the four muscles of mastication (masseter, temporalis, lateral pterygoid, and medial pterygoid) and four additional muscles: Mylohoid, Anterior belly of the Digastric, Tensor Tympani, and Tensor Veli Palatini (mnemonic = "MATT"). The masseter muscle primarily works to close the jaw, though its superficial fibers may play a limited role in protrusion of the mandible. It is the only muscle on this list of options that would con- tinue to function in facial nerve palsy. Choice A (Zygomaticus major) is incorrect. The zygomaticus major is a muscle of facial expression, so it would be paralyzed in facial nerve palsy. It functions as a dilator of the oral fissure by elevating the corners of mouth, as in smiling when the muscle contracts bilaterally or sneering to show disdain when the muscle contracts unilat- erally. It originates on the lateral aspect of the zygomatic bone, which is how it receives its name. Choice B (Levator labii superioris) is incorrect. The levator labii superioris is a muscle of facial expression, so it would be paralyzed in facial nerve palsy. It functions as a dilator of the oral fissure by retracting (elevating) the upper lip to show the upper teeth and deepens the nasolabial sulcus. It originates on the infraorbital margin of maxilla, above, and therefore covers, the infraorbital foramen. Choice C (Buccinator) is incorrect. The buccinator is a muscle of facial expression, so it would be paralyzed in facial nerve palsy. It originates on the alveolar ridges of maxillary and man- dibular molar teeth and contracts to give tension to the cheek to keep food between the occlusal surfaces of the teeth. The tone of the buccinator muscle provides resistance to keep teeth from tilting laterally and prevents patients from looking like a hamster, with food lodged in the oral vestibule, when they chew food. Choice E (Platysma) is incorrect. The platysma is a muscle of facial expression, so it would be paralyzed in facial nerve palsy. It resides in the neck and lower face to depress the mandible and wrinkle the skin of neck, as seen when a person is placed in a stressful situation. The platysma originates in the subcutaneous tissue near the clavicle and inserts into the modiolus, lateral to the labial commissures.

A 23-year-old female professional student wakes up with a facial nerve (CN VII or Bell) palsy. What muscle will continue to function despite this affliction? (A) Zygomaticus major (B) Levator labii superioris (C) Buccinator (D) Masseter (E) Platysma

D: Lingual nerve. The lingual nerve is a branch of the mandibular division of the trigeminal nerve (CN V3), which traverses the foramen ovale and resides in the infratem- poral fossa. This nerve supplies general sensation to the ante- rior two thirds of the tongue, and this nerve is at risk during extraction of an impacted third mandibular molar tooth. Choice A (Chorda tympani nerve) is incorrect. The chorda tympani, a branch of the facial nerve (CN VII), joins the lingual branch of the mandibular nerve in the infratemporal fossa. The chorda tympani nerve conveys taste sensation to the anterior two thirds of the tongue and carries presynaptic parasympa- thetic fibers to the submandibular ganglion for innervation of the submandibular and sublingual salivary glands. While the chorda tympani nerve merges with the lingual nerve to reach its effector region, cutting the chorda tympani nerve would not result in numbness on the tip of the tongue. Choice B (Mylohyoid nerve) is incorrect. The mylohyoid nerve, a branch of the mandibular division of the trigeminal nerve (CN V3), supplies motor innervation to the anterior belly of the digas- tric muscle and the mylohyoid muscles. The mylohyoid nerve branches off the inferior alveolar nerve prior to the latter nerve entering the mandibular foramen to supply sensory innervation to the inferior dentition. Cutting the mylohyoid nerve would not result in numbness on the tip of the tongue, but it would result in paralysis of the mylohyoid and anterior belly of the digastric muscles. Choice C (Inferior alveolar nerve) is incor- rect. The inferior alveolar nerve, a branch off the mandibular division of the trigeminal nerve (CN V3), enters the mandibular foramen to supply the lower teeth, periosteum, and gingivae of the mandible. The mental nerve, a terminal branch of the infe- rior alveolar nerve, supplies the skin and mucosa of the lower lip and chin. Though the inferior alveolar nerve may be dam- aged during this procedure, it is not responsible for giving gen- eral sensation to the anterior aspect of the tongue. Choice E (Glossopharyngeal nerve) is incorrect. The lingual branch of the glossopharyngeal nerve (CN IX) enters the tongue to pro- vide sensory and taste information to the posterior one third of the tongue. It also gives off a tonsillar branch, which gives sen- sation to the mucosa of the oropharynx, including the palatine tonsil. This nerve would not be responsible for the numbness on the tip of the tongue described in this patient.

A 23-year-old man has an impacted left third mandibular molar (or wisdom) tooth extracted. Following the surgery, the patient reports numbness in the anterior aspect of his tongue. Which of the following nerves is damaged? (A) Chorda tympani nerve (B) Mylohyoid nerve (C) Inferior alveolar nerve (D) Lingual nerve (E) Glossopharyngeal nerve

A. Blood accumulation between the periosteal layer of the dura and the cranial bone pulls the dura away from the bone creating a lens-shaped hyperdensity on CT

A 25-year-old man is involved in an automobile accident and slams his head into a concrete wall of a bridge. His computed tomography (CT) scan reveals a lens-shaped hyperdensity adjacent to bone. The blood creating this hyperdensity is located in which of the following spaces? A. Epidural space B. Subarachnoid space C. Subdural space D. Subpial space

B: Second division of trigeminal nerve. The second (maxillary) division of the trigeminal nerve (CN V2) supplies the skin of the inferior eyelid and upper lip through the infraorbital nerve that courses through the superior aspect (roof) of the maxillary sinus, and due to its location, this nerve is most likely damaged during the biopsy. Damage to the infraorbital nerve causes paresthesia and numbness in the areas of cutaneous (sensory) distribution for this nerve. Choice A (First division of trigeminal nerve) is incorrect. The first (oph- thalmic) division of the trigeminal nerve (CN V1) supplies the skin of the upper eyelid, forehead, and scalp. CN V1 enters the middle cranial fossa via the superior orbital fissure, and its branches are distributed within the orbit, anterior cranial fossa, and nasal cavity. CN V1 does not supply sensory innervation to the maxillary sinus, so it would be spared during this FESS. However, the ophthalmic division of the trigeminal nerve does innervate three other paranasal sinuses (ethmoidal, frontal, and sphenoidal). Choice C (Third division of trigeminal nerve) is incorrect. The third (mandibular) division of the trigeminal nerve (CN V3) supplies the skin of the lower lip, chin, cheek, anterior auricle, and aspects of the lateral scalp. CN V3 leaves the middle cranial fossa via the foramen ovale, and it does NOT have a significant branch, which courses through the superior aspect (roof) of the maxillary sinus. Therefore, it could not be damaged in this patient. Choice D (Zygomatic branch of facial nerve) is incorrect. The zygomatic branch of the facial nerve (CN VII) is a terminal branch of the main trunk of CN VII. This nerve has only motor innervation, supplying the inferior part of the orbicularis oculi and other muscles of facial expression located below the orbit. Because it does not pass through the maxillary sinus or supply cutaneous innervation to any region, the zygomatic nerve of CN VII could not cause the numbness and paresthesia seen in this patient. Choice E (Buccal branch of facial nerve) is incorrect. The buccal branch of the facial nerve (CN VII) is a terminal branch of the main trunk of CN VII. This nerve has only motor innervation, sup- plying the risorius and muscles of the upper lip. Because it does not pass through the maxillary sinus or supply cutaneous innervation to any region, the buccal nerve of CN VII could not cause the numbness and paresthesia seen in this patient.

A 24-year-old man came to his physician with a history of chronic maxillary sinusitis. A computed tomography (CT) scan reveals a soft-tissue mass in the superior aspect (or roof) of the right maxillary sinus. Functional endoscopic sinus surgery (FESS) was performed to biopsy the mass. Postoperatively, the patient experiences paresthesia and numbness of the skin of the right inferior eyelid and upper lip. Which nerve was most likely damaged during the surgery? (A) First division of trigeminal nerve (B) Second division of trigeminal nerve (C) Third division of trigeminal nerve (D) Zygomatic branch of facial nerve (E) Buccal branch of facial nerve

A: Optic nerve. Unlike other cranial nerves, the optic nerve (CN II) develops as an anterior extension of the diencephalon, part of the forebrain. Due to this unique embryological origin, the optic nerve is enveloped with exten- sion of the cranial meninges (dura, arachnoid, and pia mater) and contains cerebrospinal (CSF) fluid in its subarachnoid space. The optic nerve is formed when the axons of the retinal ganglion cells coalescence at the optic disc. A physician can view the optic disc by performing a fundoscopic examination of the back of the eye (or fundus) with an ophthalmoscope. This boxer exhibits signs of increased intracranial pressure (ICP), such as headache, nausea, and vomiting, so the phy- sician can look at the optic disc to look for papilledema, or swelling of the optic disc, which is another sign of increased ICP. If the optic disc is elevated, swollen, darker than nor- mal, or its edges are blurred, then the fundoscopic exami- nation can reveal evidence of increased ICP. These changes in the optic disc are due to the pressure transmitted via the CSF within the optic nerve. Choice B (Oculomotor nerve) is incorrect. The oculomotor nerve, CN III, innervates most of the extraocular eye muscles (with the exception of superior oblique and lateral rectus muscles) and the levator palpebrae superioris, which elevates the upper eyelid. Damage to the left CN III leads to diplopia (due to the left pupil resting in an abducted and lateral position) and ptosis (drooping of the eye- lid) associated with the left eye. However, the physician would be unable to correlate damage to the oculomotor nerve with increased ICP. Choice C (Olfactory nerve) is incorrect. The olfactory nerve, CN I, provides olfaction (or a sense of smell) to the patient. Damage to this nerve causes anosmia, or loss of olfaction, which is frequently seen in patients with severe head trauma due to the delicate olfactory nerves traversing the cribiform plate of the ethmoid bone to enter the olfactory bulb. However, damage to CN I would imply head trauma due to the punches the boxer received, not increased ICP. Choice D (Trigeminal nerve) is incorrect. The trigeminal nerve (CN V) is the main sensory nerve for the face and scalp, and it has three divisions: the ophthalmic (CN V1), maxillary (CN V2), and mandibular (CN V3). CN V3 is the only division of the trigemi- nal nerve (CN V) that supplies motor innervation. It supplies the eight muscles derived from the mesoderm of the first pha- ryngeal arch, including the muscles of mastication. Damage to the trigeminal nerve causes numbness (paresthesia) and paralysis of the muscles of mastication; however, it would not give additional information concerning increased ICP. Choice E (Trochlear nerve) is incorrect. The trochlear nerve, CN IV, innervates only one muscle, the superior oblique mus- cle, in the orbit. This muscle pulls the eye inferolaterally, but it is clinically tested by asking the patient to look inferiorly after the left eye is placed in an adducted position. Damage to this nerve would lead to diplopia, but it would not give additional information concerning increased ICP.

A 25-year-old professional boxer loses a fight when he is rendered unconscious by his opponent. After he regains consciousness, the ringside physician notes the boxer has a severe headache, nausea, and even vomiting. Being concerned about intracranial trauma, what cranial nerve can be observed by the physician, without the aid of radiographic imaging, to gain more information on whether the boxer has increased intracranial pressure? (A) Optic nerve (B) Oculomotor nerve (C) Olfactory nerve (D) Trigeminal nerve (E) Trochlear nerve

A: Buccinator. The buccinator muscle is pierced by the duct of the parotid gland as it passes toward the oral cavity to drain in the oral vestibule via a small opening opposite the second maxillary molar teeth. The muscular tone of the buccinator muscle can create a stenosis of the parotid duct, and it is the most likely site for a sialolith or calculus (L. pebble) of the parotid duct. The parotid duct arises from the anterior aspect of the parotid gland, and it courses across the superficial surface of the masseter muscle. Then, it dives medially to pierce the buccinator muscle and enters the oral cavity to drain at the second maxillary molar tooth. If sialo- lithiasis is confirmed, having a patient eat sour foods may dislodge the sialolith; however, surgical intervention may also be required to remove the calculus from the parotid duct. Choice B (Mentalis) is incorrect. The mentalis is a muscle of facial expression located on the chin that elevates and pro- trudes the lower lip or elevates the skin of the chin. It origi- nates from the incisive fossa of the mandible and contracts to show doubt in a patient. This muscle is not pierced by the duct of the parotid gland, so it plays no role in the stenosis of the parotid duct or its obstruction by the sialolith. Choice C (Temporalis) is incorrect. The temporalis is a muscle of mas- tication that elevates the mandible in closing of the mouth, and its posterior, horizontally-oriented fibers serve as the pri- mary means of retruding (retracting) the mandible when it is in a protruded (protracted) position. This muscle arises from the temporal fossa and inserts into the tip and medial border of the coronoid process of the mandible. This muscle is not pierced by the duct of the parotid gland, so it would play no role in the stenosis of the parotid duct or its obstruction by the sialolith. Choice D (Orbicularis oris) is incorrect. The orbic- ularis oris is a muscle of facial expression that encircles the mouth to act as the sphincter of the oral fissure. It originates from the incisive fossae of the mandible and maxilla and con- tracts in a patient to close the oral fissure, protrude the lips (as in kissing, whistling, or sucking), and resists distension when blowing (as seen when playing the trumpet). This muscle is not pierced by the duct of the parotid gland, so it would play no role in the stenosis of the parotid duct or its obstruction by the sialolith. Choice E (Masseter) is incorrect. The masseter is a muscle of mastication that primarily works to close the jaw, but its superficial fibers also play a limited role in protrusion of the mandible. The duct of the parotid gland does course across the superficial surface of the masseter muscle, but this muscle would play no role in the stenosis of the parotid duct because the masseter is not pierced by it.

A 35-year-old woman comes to her dentist complaining of tenderness and pain in her cheek near the parotid gland, as well as bad breath and a foul-tasting mouth at meal times. A radiopaque fluid is injected into the parotid duct system through cannulation, and this sialography of the parotid duct confirms blockage by a calculus (sialolith). What muscle, through which the parotid duct passes, is most likely causing the stenosis where the sialolith now resides, blocking the drainage of the parotid duct? (A) Buccinator (B) Mentalis (C) Temporalis (D) Orbicularisoris (E) Masseter

C. The most likely structures one would encoun- ter while performing a midline incision below the isth- mus of the thyroid gland would be the inferior thyroid vein and the thyroidea ima artery. The inferior thyroid vein drains typically to the left brachiocephalic vein, which crosses superficially, just inferior to the isthmus. The thyroidea ima artery arises from the aortic arch, vertebral artery, or other source but is not a constant structure. The middle thyroid veins drain the thyroid gland to the internal jugular vein and are superior to the incision site. The inferior thyroid arteries branch from either subclavian artery and meet the thyroid gland at an oblique angle. They would not be ligated with a midline incision. The brachiocephalic veins are inferior to the site of incision. GAS 967; GA 503-504, 515

A 35-year-old woman is admitted to the emergency department after a violent automobile crash. The patient's upper airway is obstructed with blood and mucus, and a midline tracheotomy inferior to the thyroid isthmus is performed. Which of the following vessels are most likely to be present at the site of incision and will need to be cauterized? A. Middle thyroid vein and inferior thyroid artery ⃣ B. Inferior thyroid artery and inferior thyroid vein ⃣ C. Inferior thyroid vein and thyroidea ima artery ⃣ D. Cricothyroid artery and inferior thyroid vein ⃣ E. Left brachiocephalic

D: Presynaptic parasympathetic fibers. The nerve of the pterygoid (vidian) canal consists of presynaptic parasympathetic fibers from the greater petrosal nerve of the facial nerve (CN VII) and postsynaptic sympathetic fibers from the deep petrosal nerve, derived from the periarterial arterial plexus enveloping the internal carotid artery. The presynaptic parasympathetic fibers follow the greater pet- rosal nerve (of CN VII), traverse the pterygoid canal, enter the pterygopalatine fossa to synapse in the pterygopalatine ganglion, and carry the postsynaptic parasympathetic fibers to the lacrimal gland via branches of the first two divisions of the trigeminal nerve (CN V1 and CN V2). The parasympathetic fibers of CN VII produce emotional tears from the lacrimal glands. Choice A (Taste fibers to the anterior two thirds of the tongue) is incorrect. The chorda tympani, a branch of the facial nerve (CN VII), joins the lingual branch of the man- dibular nerve (CN V3) in the infratemporal fossa. The chorda tympani nerve conveys taste sensation from the anterior two thirds of the tongue and carries presynaptic parasympathetic fibers to the submandibular ganglion for innervation of the submandibular and sublingual salivary glands. However, the chorda tympani nerve is not associated with the pterygoid canal, so it would not be damaged by a facial nerve schwan- noma at this location. Choice B (Parasympathetic innervation to the submandibular and sublingual glands) is incorrect. The chorda tympani, a branch of the facial nerve (CN VII), joins the lingual branch of the mandibular nerve (CN V3) in the infratemporal fossa. The chorda tympani nerve conveys taste sensation to the anterior two thirds of the tongue and carries presynaptic parasympathetic fibers to the submandibular gan- glion for innervation of the submandibular and sublingual salivary glands. However, the chorda tympani nerve is not associated with the pterygoid canal, so it would not be dam- aged by a facial nerve schwannoma at this location. Choice C (Presynaptic sympathetic fibers) is incorrect. All sympathetic fibers in the head are postsynaptic because they synapse in the superior cervical ganglion before reaching their targets in the head. Therefore, the sympathetic fibers traveling in the pterygoid canal are postsynaptic. These postsynaptic sym- pathetic fibers control the smooth muscle tone (vasodilation and vasoconstriction) of the blood vessels lying underneath the nasal mucosa. Choice E (Postsynaptic parasympathetic fibers) is incorrect. The parasympathetic fibers derived from the greater petrosal nerve (of CN VII) do not synapse until after they have left the pterygoid (vidian) canal. After leaving this canal, these presynaptic parasympathetic fibers synapse in the pterygopalatine ganglion distal to the pterygoid canal. Therefore, the parasympathetic fibers in the pterygoid (vidian) canal are presynaptic. These fibers serve to increase emotional tearing of the lacrimal glands.

A 37-year-old man presented with decreased emotional tear- ing on the right side and intermittent headaches. Magnetic resonance images (MRIs) revealed a facial nerve schwannoma located within the right pterygoid (vidian) canal. What nerve fibers are most likely injured by this tumor? (A) Taste fibers to the anterior two thirds of the tongue (B) Parasympathetic innervation to the submandibular and sublingual glands (C) Presynaptic sympathetic fibers (D) Presynaptic parasympathetic fibers (E) Postsynaptic parasympathetic fibers

A after innervating the SCM, the spinal accessory nerve (CN XI) crosses the posterior triangle of the neck to pass deep to the trapezius muscle

A 40-year-old male is undergoing surgical dissection of the posterior triangle of the neck to remove an enlarged lymph node. Which of the following structures is at greatest risk during this surgery? A. Spinal accessory nerve B. Superior thyroid artery C. Thoracic duct D. Vagus nerve E. Vertebral artery

C Rupture of the middle meningeal artery in the cranial cavity results in the accumulation of blood between the dura and cranial bone. In the cranial cavity the middle meningeal artery is located in the vicinity of pterion.

A 40-year-old unconscious man is admitted to the emergency department after being hit in the head with a baseball. A CT scan reveals a fractured pterion and an epidural hematoma. Laceration of branches of which of the following arteries accounts for the epidural hematoma? A Superficial temporal B Supraorbital C Middle meningeal D Middle cerebral of internal carotid E Vertebral

A Structures of the eye that receive parasympathetic include the ciliary muscles and the constrictor pupillae.

A 41-year-old woman overdoses on some prescription medications that have a common side effect of autonomic nerve stimulation. Which of the following conditions or actions results from stimulation of the parasympathetic fibers to the eye? A Contraction of the ciliary muscles B Dilation of the pupil C Flattening of the lens D Vasoconstriction of the capillaries of the iris

A: Internal acoustic meatus. The loss of facial expression and drooping corner of the mouth indi- cate paralysis of the facial muscles and damage to the facial nerve (CN VII). The intact ability to clench the jaw and chew denotes proper functioning of the muscles of mastication and an intact mandibular division of the trigeminal nerve (CN V3). The hearing loss and unsteady balance indicate failure in the inner ear complex implicating the right ves- tibulocochlear nerve (CN VIII; auditory nerve). The absence of the blink reflex is related to loss of the orbicularis oculi muscle, the facial muscle responsible for closing the eye- lids, and this evidence reinforces a problem with the facial nerve. Normal cutaneous sensation across the face indicates the entire trigeminal pathway is intact. Therefore, the sus- pected tumor affects both CN VII and CN VIII, but not the trigeminal nerve (CN V). The only location where CN VII and CN VIII can be affected simultaneously is at the internal acoustic meatus (in the wall of the posterior cranial fossa), where the paired nerves leave the cranial cavity to enter the petrous part of the temporal bone. Very quickly thereafter, the nerves diverge and follow separate pathways to their target regions. The given contrast-enhanced T1-weighted MRI demonstrates a right-sided vestibular schwannoma (acoustic neuroma), located at the internal acoustic meatus and identified by the arrow, which confirms this diagnosis. This vestibular schwannoma, which is clearly seen in white due to the gadolinium-based intravenous contrast, resides at the cerebellopontine angle and affects the facial and ves- tibulocochlear nerves as they emerge from this location. This tumor would also increase intracranial pressure potentially causing pontomedullary brain stem compression. Choice B (Foramen ovale) is incorrect. The mandibular division of the trigeminal nerve (CN V3) passes through the foramen ovale in the floor of the middle cranial fossa. At this location, a tumor would affect the muscles of mastication and cutane- ous sensation over the mandibular region of the face, which is not evident in this patient. Choice C (Foramen rotundum) is incorrect. The maxillary division of the trigeminal nerve (CN V2) passes through this opening in the anterior wall of the middle cranial fossa. Nerve damage here would affect cutaneous sensation across the midfacial region, which is not evident in this patient. Choice D (Geniculum of the facial canal) is incorrect. The facial nerve travels through the facial canal within the petrous part of the temporal bone, including the knee-like bend (geniculum) of the canal. Thus, tumor growth in this location would affect the facial nerve and pro- duce the facial paralysis described in this case. However, the vestibulocochlear nerve would not be affected at this site, as it has already separated from the facial nerve. Choice E (Stylomastoid foramen) is incorrect. The main branch of the facial nerve exits the skull through the stylomastoid foramen at the base of the skull. Tumor growth here would affect the facial nerve, producing the described facial paralysis. How- ever, the vestibulocochlear nerve would not be affected by problems at this location.

A 43-year-old man presents with loss of control of facial expression across the entire right side. The corner of his mouth droops on the right side, but he can clench his jaw and chew on demand. During examination, his physician also notes loss of hearing on the right side, and the patient has dif- ficulty maintaining balance while standing on one foot. The patient's corneal (blink) reflex is absent in the right eye, but cutaneous sensation is normal on the entire face. The physician orders radiographic imaging in anticipation of finding a tumor. What is the most likely location of the tumor? (A) Internal acoustic meatus (B) Foramen ovale (C) Foramen rotundum (D) Geniculum of the facial canal (E) Stylomastoid foramen

C. The superior salivatory nucleus is the autonomic nucleus for the facial nerve. Parasympathetic fi bers carried by the greater petrosal branch of the facial nerve are responsible for supply of the lacrimal gland and sinuses, via the pterygopalatine ganglion. The geniculate ganglion contains the cell bodies for taste from the anterior two thirds of the tongue carried by the chorda tympani branch of the facial nerve. This branch also carries the parasympathetic supply for the submandibular and sublingual salivary glands. The auriculotemporal nerve provides sensory innervation to the temporal regions of the head, the TMJ, and general sensation from the ear. The inferior salivatory nucleus provides preganglionic parasympathetic fi bers carried by the glossopharyngeal nerve that synapse in the otic ganglion, providing parotid stimulation. The pterygopalatine ganglion includes fi bers that innervate only lacrimation and the nasal sinuses, but not taste on the anterior two thirds of the tongue.

A 45-year-old female is admitted to the hospital with severe headaches, dizziness, and vomiting. Radiographic examination reveals an intracranial tumor. Upon physical examination the patient has dryness of the nasal and paranasal sinuses, loss of lacrimation, and loss of taste from the anterior two thirds of the tongue. Which of the following structures is most likely involved with the tumor? A. Auriculotemporal nerve ⃣ B. Lesser petrosal nerve ⃣ C. Facial nerve ⃣ D. Inferior salivatory nucleus ⃣ E. Pterygopalatine ganglion

A, D A and D are both correct. With only the superior oblique and lateral rectus functional, the eye will be abducted (lateral rectus), and slightly depressed and medially rotated (superior oblique). However, medial rotation will not be evident on observation alone.

A 45-year-old female presents to her primary care physician with complaints of a droopy right eyelid, double vision, an increased sensitivity to light, and difficulty reading at a close distance. Her physician suspects a cranial nerve injury. In evaluating this patient the physician notes which of the following presentations of the affected eye when in primary position? (More than one answer possible) A The pupil of the eye is abducted B The pupil of the eye is adducted C The pupil of the eye is slightly elevated D The pupil of the eye is slightly depressed

B: Inferior oblique. Paralysis of one or more of the extraocular muscles causes a lack of coordinated eye movements, often resulting in diplopia (double vision). Clini- cal evaluation of the muscles includes step-wise positioning of the eye in such a way as to test each individual muscle at its position of greatest mechanical efficiency (relative to the visual axis) to determine if that muscle is functioning prop- erly. Testing begins from the rest position, with the patient looking straight ahead. In this case, the first step (looking inward; adduction) tests the action of the medial rectus, the primary adductor of the eye. At the adducted position, the superior and inferior oblique muscles are aligned along their primary lines of action. The second step (looking upward; elevation) tests for function of the inferior oblique, which elevates the eye. Failure of either step indicates possible dam- age to the oculomotor nerve (CN III), which supplies both of these muscles. The inferior oblique and inferior rectus muscles can be impinged in head trauma cases when the floor of the orbit is fractured. These orbital (blowout) frac- tures lead to the downward herniation of orbital contents into the maxillary sinus, leading to the potential entrapment of these two extraocular muscles. The integrity of the infe- rior oblique muscle was tested in this question. Choice A (Superior oblique) is incorrect. Testing the superior oblique includes the same first step as for the inferior oblique, that is, adducting the eye to test the medial rectus and to position the oblique muscles along their primary lines of action. In this case, the second step is to look downward (depression) to test the superior oblique, which depresses the eye. Failure of this step indicates possible damage to the trochlear nerve (CN IV), which supplies only the superior oblique muscle. Choice C (Lateral rectus) is incorrect. The lateral rectus is the primary abductor of the eye from the rest position. Asking the patient to look outward (away from the nose) tests the ability of the lateral rectus to abduct at its position of greatest efficiency. Failure of this test indicates possible damage to the abducent nerve (CN VI), which supplies only the lateral rectus. Fur- thermore, at the adducted position, the superior and inferior rectus muscles are aligned along their primary lines of action and are set in best testing position. Choice D (Inferior rectus) is incorrect. At the abducted position, asking the patient to look downward tests the function of the inferior rectus, which depresses the eye. Failure to accomplish this task indicates possible damage to the oculomotor nerve, which supplies the inferior rectus muscle. Choice E (Superior rectus) is incor- rect. From the abducted position, asking the patient to look upward tests the superior rectus, which elevates the eye. Fail- ure to accomplish this task is another indicator of possible damage to the oculomotor nerve, which supplies the superior rectus muscle.

A 47-year-old man has trouble with double vision (diplopia) after striking his head on the steering wheel in a car accident. During a subsequent eye examination, his ophthalmol gist asks him to first look inward (toward his nose) and then upward (toward the ceiling). The integrity of which of the following extraocular muscles is being tested? (A) Superioroblique (B) Inferior oblique (C) Lateralrectus (D) Inferiorrectus (E) Superior rectus

D. The chin and lower lip area are supplied by the mental nerve, a branch of the inferior alveolar nerve, which in turn is a branch of the mandibular division of the trigeminal nerve. The auriculotemporal nerve supplies the TMJ, the temporal region, the parotid gland, and the ear. The buccal nerve is sensory to the internal surface of the cheek. The lesser petrosal nerve is a parasympathetic nerve and would not be affected by herpes zoster, a disease of the dorsal root ganglia. The infraorbital nerve provides sensory innervation to the upper lip.

A 50-year-old woman complained of pain over her chin and lower lip. A few days later small vesicles appeared over the same area and soon began erupting. She was diagnosed with a dermatomal herpes zoster inflammation (shingles). Which of the following nerves was most likely responsible for the transmission of the virus in this case? A. Auriculotemporal ⃣ B. Buccal ⃣ C. Lesser petrosal ⃣ D. Mental ⃣ E. Infraorbital

E: Sphenopalatine. The sphenopalatine artery supplies most of the blood to the nasal cavity, particularly the inferior and posterior aspects of the nasal cavity. Therefore, it is highly probable that the sphenopalatine artery, a terminal branch of the maxillary artery, was the source of the epistaxis (nosebleed) in this patient, as seen in the given illustration located on the next page. Choice A (Greater palatine) is incor- rect. The greater palatine artery supplies blood primarily to the hard palate; however, it may supply a small amount of blood to the anterior and inferior aspects of nasal cavity via its com- munication with the sphenopalatine artery through the inci- sive canal. However, due to the location of the injury in this patient, the greater palatine artery would not be involved with this persistent nosebleed. Choice B (Infraorbital) is incorrect. The infraorbital artery courses in the roof of the maxillary sinus to give blood to this area as well as the superior canine and incisor teeth, inferior aspect of the orbit, and superior aspect of the lip. The infraorbital artery would not be responsible for the nosebleed seen in this patient. Choice C (Facial) is incorrect. The lateral nasal branch of the facial artery would supply the anterior and inferior aspects of the nasal cavity in the region of the vestibule; however, bleeding of this artery would be easily stopped by applying pressure to the alae of the nose. Choice D (Anterior ethmoidal) is incorrect. The anterior ethmoidal artery supplies the anterior, superior aspect of the nasal cavity after entering the nasal cavity through the cribi- form plate of the ethmoid bone. This artery does not supply blood to posterior and inferior aspect of the lateral nasal wall, which is the location of the bleed in this patient.

A 65-year-old man complains of a persistent nosebleed. His physician uses a cotton swab to apply pressure at the source, the inferior and posterior aspects of the lateral nasal wall. Which artery is the most likely source of the bleeding? (A) Greaterpalatine (B) Infraorbital (C) Facial( D) Anteriorethmoidal (E) Sphenopalatine

D. The pharyngotympanic (eustachian) tube connects the middle ear and the nasopharynx and is the conduit for spreading infections. The choanae are the openings between the nasal cavity and the nasopharynx, but they are not involved in spreading infection. The internal and external auditory meatuses are not directly associated with the middle ear but are associated with the inner and outer ear, respectively. The pharyngeal recess is a slitlike opening located behind the entrance to the auditory tube in the nasopharynx. Adenoids, enlarged masses of lymphoid tissue, can develop there.

A 7-year-old boy with a high fever is brought to the pediatrician. During physical examination the patient complains of pain in his ear. His throat appears red and infl amed, confi rming the diagnosis of pharyngitis. Which of the following structures provided a pathway for the infection to spread to the tympanic cavity (middle ear)? ⃣ A. Choanae ⃣ B. Internal acoustic meatus ⃣ C. External acoustic meatus ⃣ D. Pharyngotympanic tube ⃣ E. Pharyngeal recess

C: Levator palpebrae superioris. The levator palpebrae superioris muscle attaches into the tarsal plate and skin of the upper eyelid and is the primary elevator of the eye- lid. Weakness in this muscle results in ptosis (drooping) of the upper eyelid and may reflect a problem with the oculomotor nerve (CN III). The oculomotor nerve provides the motor con- trol for the levator palpebrae superioris and most (4 of 6) of the extraocular muscles. The levator palpebrae superioris is assisted by the superior tarsal muscle, a thin smooth muscle sheet in the upper eyelid innervated by sympathetic fibers. Choice A (Orbicularis oculi) is incorrect. The orbicularis oculi is a muscle of facial expression that acts to close the eye, form- ing a sphincter-like arrangement around the orbit and extend- ing into the eyelids. Functional deficits of the orbicularis oculi muscle result in loss of the ability to blink, which endangers the health of the eye by hampering proper spread of tears across the eyeball. Choice B (Frontalis) is incorrect. The fron- talis muscle is the anterior component of the epicranius muscle in the scalp. It elevates the eyebrows and produces the hori- zontal wrinkles across the forehead, occurring when a person looks up (superior). Choice D (Superior rectus) is incorrect. The superior rectus muscle is one of the extraocular muscles. It attaches onto the sclera on the superior aspect of the eye and acts as the primary elevator of the eyeball. Choice E (Orbital muscle) is incorrect. The orbital muscle is a rudimentary smooth (nonstriated) muscle sling across the orbit that helps to support and position the eyeball within the orbit.

A 75-year-old man tells his physician he has been having pro- gressively more trouble opening his left eye because his upper eyelid tends to droop. Which of the following muscles is most likely weakened? (A) Orbicularis oculi (B) Frontalis (C) Levator palpebrae superioris (D) Superior rectus (E) Orbital muscle

B: Retropharyngeal space. The retropharyngeal space is the most frequent route for infection to spread through the neck into the superior mediastinum. This potential space exists between the prevertebral fascia and the anterior lamina of the prevertebral fascia, and it extends from the base of the skull to the superior mediastinum to the level of the third tho- racic vertebra. The retropharyngeal space allows movement of the esophagus, pharynx, larynx, and trachea relative to the vertebral column, but it is clinically important due to its poten- tial to provide a conduit for the spread of infection into the mediastinum. Choice A (Parapharyngeal space) is incorrect. The parapharyngeal space is located between the lateral wall of the upper pharynx, the medial pterygoid muscle, and the cervical vertebrae. An infection residing in this potential space is unable to reach the superior mediastinum, unless it com- municates directly with the retropharyngeal space. Choice C (Buccal space) is incorrect. The buccal space exists between the deep surface of the parotid gland and the mucosa of the cheek. An infection residing in this potential space is unable to reach the superior mediastinum. Choice D (Carotid sheath) is incorrect. The carotid sheath is a fascial investment that extends from the base of the skull to the root of the neck, and it does communicate with the mediastinum of the thorax. So, this fascial space does offer a potential pathway for the spread of infection into the mediastinum; however, based upon the source of the infection in the tonsillar fossa, the retropha- ryngeal space is the best answer to this question. Choice E (Suprasternal space) is incorrect. The suprasternal space is located above the manubrium of the sternum, and it is a nar- row interval located between a split in the investing layer of the deep cervical fascia. This space would allow a communica- tion between the anterior jugular veins to pass, but it would not allow an infection to spread through the neck and into the mediastinum.

A 9-year-old girl with a history of strep throat has her palatine tonsils surgically removed. She returns to the hospital 3 days later with a high fever and chest pain. A physician orders a CT scan, which revealed spread of infection into the superior mediastinum. What is the most likely route for this infection to descend through the neck to reach the superior mediastinum? (A) Parapharyngeal space (B) Retropharyngeal space (C) Buccal space (D) Carotid sheath (E) Suprasternal space

E: Left oculomotor nerve. The left oculomotor nerve (CN III) is damaged in this patient. The physician is performing the pupillary light reflex, which tests the integrity of the sensory and motor functions of the eye. The afferent limb of the reflex is the optic nerve, and the efferent limb is the oculomotor nerve. In this patient, the constriction of the right pupil (a consensual response to the light) implies the affer- ent limb (left optic nerve) of the light reflex is intact because one of the pupils responded to the light. However, the effer- ent limb of the left eye is likely damaged due to the lack of a direct response (pupillary constriction) to the light. Under normal circumstances, both pupils constrict in response to increased light intensity due to a bilateral projection from the pretectal nucleus within the upper medulla to the Edinger- Westphal nucleus, which then projects its parasympathetic fibers along the oculomotor nerve causing pupillary constric- tion. Choice A (Right optic nerve) is incorrect. Because the physician did not direct the light into the right eye, the integ- rity of the right optic nerve was not tested. To test this afferent limb of the pupillary light reflex, the physician must shine the light directly into the right eye. Because there was no direct response (pupillary constriction) of the left eye, the physician did confirm damage to the left oculomotor nerve. Choice B (Left optic nerve) is incorrect. The left optic nerve is intact in this patient because the right pupil constricted (a consensual response), which implies the patient's left optic nerve recog- nized the increase in light intensity. Choice C (Right ophthal- mic nerve) is incorrect. The ophthalmic nerve (or first division of the trigeminal nerve) supplies sensory (cutaneous) innerva- tion to the skin of the upper eyelid, cornea, anterior aspect of the nose, forehead, and anterior scalp. This nerve did not participate in the pupillary light reflex, so this option can be eliminated. Choice D (Right oculomotor nerve) is incorrect. The right oculomotor nerve is intact because the right pupil constricted in response to the increased luminescence. The bilateral projections within the pupillary light response path- way enable this consensual response.

A physician directs a small light into only the left eye of a patient to test pupillary constriction. The left pupil does not respond to the light; however, the right pupil constricts. What nerve is most likely damaged in this patient? (A) Right optic nerve (B) Left optic nerve (C) Right ophthalmic nerve (D) Right oculomotor nerve (E) Left oculomotor nerve

F The hypoglossal nerve innervates the muscles of the tongue. Loss of function of the left genioglossus muscle of the tongue will result in the tongue deviating to the left upon protrusion.

A routine physical exam of a 39-year-old female patient reveals deviation of her tongue to the left on protrusion. Which of the following nerves is injured? A Right glossopharyngeal nerve (CN IX) B Left glossopharyngeal nerve (CN IX) C Right vagus nerve (CN X) D Left vagus nerve (CN X) E Right hypoglossal nerve (CN XII) F Left hypoglossal nerve (CN XII)

The answer is E: Facial nerve. The tympanic (middle ear) cavity is a small air-filled space within the petrous part of the temporal bone. Its position, shape, and relations make for challenging spatial concepts in anatomy because it is buried so deeply within the skull. It has six walls: the roof (tegmental wall), floor (jugular wall), anterior (carotid) wall, posterior (mastoid) wall, medial (labyrinthine) wall, and lateral (membranous) wall. Each wall has a close relationship to one or more significant neighboring structures. The upper part of the posterior wall contains an opening (aditus) that leads to the mastoid antrum and air cells. The distal limb of the facial canal (containing the main branch of the facial nerve) descends below the aditus, behind the posterior wall, on its way to its termination at the stylomastoid foramen. Thus, a tumor pierc- ing the posterior wall may invade the facial canal and/or the mastoid air sinuses. Choice A (Internal jugular vein) is incor- rect. The superior bulb of the internal jugular vein lies beneath the floor (jugular wall) of the tympanic cavity. Because this tumor is embedded in the posterior wall of the middle ear cav- ity, the internal jugular vein would not be affected. Choice B (Tympanic membrane) is incorrect. The tympanic membrane (eardrum) forms most of the lateral (membranous) wall. The epitympanic recess, situated above the tympanic membrane, completes the lateral wall. The handle of the malleus and the crossing chorda tympani nerve lie against the inside of the tympanic membrane. Because this tumor is embedded in the posterior wall of the tympanic cavity, the tympanic membrane would not be affected. Choice C (Internal carotid artery) is incorrect. The carotid canal (containing the internal carotid artery) lies outside the anterior (carotid) wall. Also, the phar- yngotympanic (auditory) tube and the canal for the tensor tympani muscle open into the upper part of the anterior wall. Because this tumor is embedded in the posterior wall of the tympanic cavity, the internal carotid artery would not be affected. Choice D (Brain) is incorrect. The roof (tegmental wall) of the tympanic cavity is the tegmen tympani (roof of tympanum). This thin bony layer forms part of the floor of the middle cranial fossa, separating the dura mater and temporal lobe of the brain from the middle ear. Thus, fractures here may result in leakage of cerebrospinal fluid into the tympanic cav- ity and subsequently into the nasopharynx (via the pharyngo- tympanic tube) or external acoustic meatus (if the tympanic membrane is ruptured). The sixth wall (medial; labyrinthine) separates the middle ear from the internal ear. Its main fea- tures are the promontory, oval window, and round window.

A tumor is discovered embedded in the posterior wall of the tympanic cavity in a 45-year-old man. If the tumor erodes through this wall, which of the following structures will it first encounter? (A) Internal jugular vein (B) Tympanic membrane (C) Internal carotid artery (D) Brain (E) Facial nerve

B: Abducent nerve. All of the listed nerves travel within the cavernous sinus; however, the abducent nerve (CN VI) parallels the course of the internal carotid artery within the cavernous sinus, which makes it the most likely nerve to be damaged. The internal carotid artery and abducent nerve are located medially as they traverse the cavernous sinus. In the given figure, the other nerves that travel through the cavern- ous sinus (CNs III, IV, V1, and V2) lie laterally. Therefore, the increased pressure, seen within the internal carotid artery due to the atherosclerotic plaque causing the stenosis and result- ing aneurysm, would most likely affect the abducent nerve. Choice A (Trochlear nerve) is incorrect. The trochlear nerve (CN IV) traverses the cavernous sinus; however, it lies along the lateral wall of the sinus. The abducent nerve, which paral- lels the internal carotid artery, would most likely be affected by increased pressure within this vessel due to the stenosis and subsequent aneurysm. Choice C (Maxillary nerve) is incor- rect. The maxillary nerve (or second division of the trigeminal nerve) traverses the cavernous sinus; however, it lies along the lateral wall of the sinus. The abducent nerve, which parallels the internal carotid artery, would most likely be affected by increased pressure within this vessel due to the stenosis and subsequent aneurysm. Choice D (Ophthalmic nerve) is incor- rect. The ophthalmic nerve (or first division of the trigeminal nerve) traverses the cavernous sinus; however, it lies along the lateral wall of the sinus. The abducent nerve, which paral- lels the internal carotid artery, would most likely be affected by increased pressure within this vessel due to the stenosis and subsequent aneurysm. Choice E (Oculomotor nerve) is incorrect. The oculomotor nerve (CN III) traverses the cavern- ous sinus; however, it lies along the lateral wall of the sinus. The abducent nerve, which parallels the internal carotid artery, would most likely be affected by increased pressure within this vessel due to the stenosis and subsequent aneurysm.

An MRI of the right internal carotid artery reveals atheroscle- rotic plaques causing stenosis of the vessel's lumen within the cavernous sinus. The stenosis is causing increased pressure within the internal carotid artery as it courses through the cavernous sinus, resulting in an aneurysm. Given its location, what cranial nerve would most likely be damaged? (A) Trochlearnerve (B) Abducent nerve (C) Maxillary nerve (D) Ophthalmic nerve (E) Oculomotor nerve

D The motor portion of the facial nerve exits the stylomastoid foramen and enters the substance of the parotid gland before distributing to the muscles of facial expression located in the face. With the exception of a small number of sensory fibers that innervate skin of the external ear, the motor portion of the facial nerve carries only motor fibers.

An injury to the nerve exiting the stylomastoid foramen would disrupt which of the following functions? A Sensation to the anterior scalp B Sensation to the cheek C Secretion of the parotid gland D Closing of the eyelid

B: Opening of the parotid duct. The vestibule of the mouth is the narrow space between the cheeks and lips and the hard wall of teeth and related gingivae (gums). The duct of the parotid gland drains into the vestibule, opposite the upper (maxillary) second molar tooth. The opening of the parotid duct can be visualized in an oral examination and readily located by the tongue. The integrity of the wall of the oral ves- tibule is critical in food handling and articulate speech. Also, the vestibule is important in oral hygiene as food and other materials may collect and fester there if the area is not cleaned properly. Choice A (Lingual frenulum) is incorrect. The lin- gual frenulum is a thin, midline fold of tissue that anchors the underside of the tongue to the floor of the oral cavity proper. Abnormal shortening of the frenulum results in ankyloglossia (tongue-tie). In this condition, the tongue has limited mobil- ity, which may impact food handling and speech, potentially producing a speech impediment. Choice C (Opening of the submandibular duct) is incorrect. The submandibular duct runs forward across the oral floor and drains at the sublingual papilla (caruncle) on the side of the lingual frenulum. Choice D (Uvula) is incorrect. The uvula is the soft midline extension of the soft palate that is most often identified as "that thing that hangs down in the back of the mouth." It serves to assist in sealing off the nasopharynx during swallowing so that food does not regurgitate into the nasal passages. Deviation of the uvula during phonation (saying "Ah") indicates lesion of the vagus nerve (CN X), which innervates the muscle of the uvula (musculus uvulae). Choice E (Palatine tonsil) is incorrect. The palatine tonsil lies on the side of the oropharynx, between the palatoglossal and palatopharyngeal arches. It is highly vascular and may bleed profusely during tonsillectomy.

As part of an initial oral examination of a new patient, a dental hygienist inspects the vestibule of the mouth. Which of the following structures is encountered in this area? (A) Lingualfrenulum (B) Opening of the parotid duct (C) Opening of the submandibular duct (D) Uvula (E) Palatine tonsil

E Tilting of the head is seen in patients with an injury of the trochlear nerve and the associated weakness or paralysis of the superior oblique muscle it innervates. Dysfunction of the superior oblique muscle results in unopposed contraction of the inferior oblique muscle, its primary antagonist, and lateral rotation of the affected eye. Tilting the head away from the side of the nerve deficit medially rotates the unaffected eye to bring the two eyes into better alignment. Grandfather's tilting of his head towards his right shoulder indicates the lesion is on the left side.

At Thanksgiving dinner you notice for the first time that your grandfather always holds his head at a slight angle when looking directly at you. Suspecting an extra-ocular muscle deficit of some kind, you seize this opportunity to apply your newly-gained knowledge of eye movements and clinical testing to diagnose his condition. In your initial observation of your "patient" you notice that when looking straight ahead your grandfather's head is tilted towards his right shoulder. Assuming this is a cranial nerve injury, which of the following cranial nerves is likely affected? A Left abducens B Right abducens C Left oculomotor D Right oculomotor E Left trochlear F Right trochlear

B: Greater wing of the sphenoid bone. The pte- rion is a significant craniometric landmark point in the tem- poral fossa on the lateral aspect of the skull. It is the roughly H-shaped junction of four bones: greater wing of the sphe- noid, frontal, parietal, and squamous parts of the temporal. The pterion is clinically significant in that the bones here are relatively thin and susceptible to fracture from impact. Fur- thermore, the anterior branch of the middle meningeal artery typically lies tightly grooved against the interior of the skull at this point. The middle meningeal artery is the major vessel supplying the dura mater and the bones of the cranial vault. It can be readily ruptured in trauma to the pterion, and the middle meningeal artery is the primary vessel implicated in epidural hemorrhage. Choice A (Zygomatic process of the temporal bone) is incorrect. The zygomatic process of the temporal bone is an anterior projection articulating with the zygomatic bone. Together, these two components form the zygomatic arch. Choice C (Mastoid process of the temporal bone) is incorrect. The mastoid process of the temporal bone is a breast-like projection extending inferior from the tempo- ral bone behind the external acoustic meatus. It forms part of the posterior boundary of the infratemporal fossa. Addition- ally, it is the attachment site for certain neck muscles (e.g., sternocleidomastoid) and is largely hollowed by the mastoid air cells. Choice D (Lateral pterygoid plate of the sphenoid bone) is incorrect. The lateral pterygoid plate is a thin, wing- like, inferior extension of the sphenoid bone. It forms the medial (deep) boundary of the infratemporal fossa and pro- vides attachment for portions of the medial and lateral ptery- goid muscles. Choice E (Coronoid process of the mandible) is incorrect. The coronoid process of the mandible is a thin, blade-like, superior projection from the anterior aspect of the ramus of the mandible. It serves as the insertion area for the large temporalis muscle, a muscle of mastication.

During a fight between two construction workers, one man strikes the other with a hammer at the pterion of the skull. Which of the following bones may be fractured? (A) Zygomatic process of the temporal bone (B) Greater wing of the sphenoid bone (C) Mastoid process of the temporal bone (D) Lateral pterygoid plate of the sphenoid bone (E) Coronoid process of the mandible

D injury to the hypoglossal n. in the periphery will weaken the IPSILATERAL genioglossus muscle, causing the tongue to deviate to that same side on protrusion

Following a cervical carotid endartectomy performed to remove a large atherosclerotic plaque it is noted that the patient's tongue deviates to the right when asked to protrude it. Which of the following nerves was most likely injured during the procedure? A. Left glossopharyngeal B. Right glossopharyngeal C. Left hypoglossal D. Right hypoglossal E. Left vagus F. Right vagus

coronal suture The arrows are at a line running between the frontal and parietal bones. This is the coronal suture.

Idenfity the feature indicated by the arrows.

Crista Galli The arrow is directed at a bony formation superior to the nasal cavity and in the midline. The cribriform plate forms the superior limit of the nasal cavity in the midline. The crista galli is a vertical superior projection of bone from that plate. While this individual has a well formed septum of the frontal sinus and it is also in this location it is not one of your objectives on radiographs.

Identify the bony feature indicated.

angle of mandible Although this is superimposed on other bone you can still follow the outline of the mandible. Where the mandible changes direction is the angle of the mandible.

Identify the bony feature that is outlined.

foramen magnum The question asked for opening. So even though the dens is also encircled in this image the correct answer is foramen magnum.

Identify the opening outlined.

B: Dense connective tissue. The second layer of the scalp is composed of dense fibrous connective tissue, and this layer houses the main networks of nerves and vessels that enter the scalp from its periphery. The walls of the arteries within the scalp are tightly attached to the surrounding con- nective tissue bed. As a result, scalp lesions that penetrate the second layer tend to bleed profusely because the fibrous tissue holds apart the cut ends of the severed vessels. Conveniently, the scalp can be divided into five structural/functional layers, which create a mnemonic "SCALP" when moving from super- ficial to deep zones. Layer 1 (most superficial) is Skin. Layer 2 is the (dense) Connective tissue highlighted in this question. Layer 3 is Aponeurosis of the epicranius muscle (or the epicra- nial aponeurosis). Layer 4 is Loose (areolar) connective tissue that enables free movement of the first three layers of the scalp over the calvaria. Layer 5 is Pericranium (external periosteum), which is a dense layer of connective tissue fused to the outer surfaces of the neurocranium. The mnemonic "SCALP" can enable a student to recall the five layers of the scalp. Choice A (Skin) is incorrect. Skin is the first layer of the scalp, and it is the typically hair-bearing epidermis rich in sebaceous glands. Choice C (Epicranial aponeurosis) is incorrect. The epicra- nius (occipitofrontalis) muscle is formed by the frontalis and occipitalis muscles and the expansive aponeurosis that binds them together. This unit forms the third layer of the scalp. It is responsible for movement of the scalp by the actions of its muscular ends. Deep scalp wounds through the epicranius (especially the aponeurotic part in the frontal plane) tend to gape widely because the frontalis and occipitalis muscles pull in opposite directions and widen the lesion. Choice D (Loose connective tissue) is incorrect. Loose (areolar) connective tissue is the fourth layer of the scalp, and it forms the sub- aponeurotic space of the scalp, which becomes the plane of movement enabling the first three layers of the scalp to slide as a unit relative to the underlying pericranium. This loose con- nective tissue is also considered the danger layer of the scalp because infectious matter (blood, pus) can spread easily and widely through this fourth layer of the scalp. Also, infections can move from this layer through emissary (epiploic) veins into the intracranial dural venous sinuses. Choice E (Pericra- nium) is incorrect. The deepest (fifth) layer of the scalp is the pericranium (or external periosteum), which is a dense layer of connective tissue fused to the outer surfaces of the bones of the cranial vault.

Startled by a loud noise while shaving his head with a straight razor, a young man accidentally cuts his scalp, severing branches of the supraorbital and superficial temporal vessels. The scalp wound appears modest but bleeds profusely. In what layer of the scalp do the severed vessels reside? (A) Skin (B) Dense connective tissue (C) Epicranialaponeurosis (D) Loose connective tissue (E) Pericranium

A Cell bodies of all preganglionic autonomic neurons (sympathetic and parasympathetic) are located in the CNS. The cell bodies of the preganglionic parasympathetic neurons that innervate head structures are located in various nuclei in the brainstem.

The cell bodies of the preganglionic parasympathetic neurons that innervate effector organs in the head are located in which of the following structures? A Brainstem B Superior cervical ganglion C Dorsal root ganglion D pterygoapalatine ganglion

D The cervical plexus is said to be formed from the ventral primary rami of either the C1-C4 spinal nerves, or the C1-C5 spinal nerves. The difference in interpretation depends upon whether the phrenic nerve (C3-C5) is considered a branch of the cervical plexus. In this course we do consider the phrenic nerve a branch of the cervical plexus.

The cervical plexus is formed from the ventral rami of which of the following spinal nerves? A C1-C8 B C3-C8 C C2-C6 D C1-C5

A,C,D,F

Which of the following ganglia houses the cell bodies of postganglionic parasympathetic neurons? (select all that apply) A Ciliary B Dorsal root C Otic D Pterygopalatine E Stellate F Submandibular

B,C,D E

Which of the following structures is embedded in the dural wall of the cavernous sinus? (select all that apply) A CN II B CN III C CN IV D CN V1 E CN V2 F CN V3 G CN VI H Internal carotid artery I Ophthalmic artery

E: Ophthalmic division of trigeminal. The oph- thalmic (first) division of trigeminal nerve (CN V1) supplies sensory (cutaneous) innervation to the skin of the upper eye- lid, anterior aspect of the nose, forehead, and anterior scalp. CN V1 enters the middle cranial fossa through the superior orbital fissure, and its three terminal branches (lacrimal, naso- ciliary, and frontal nerves) are distributed throughout the orbit, anterior cranial fossa, anterior scalp, and nasal cavity. Other cranial nerves traverse the superior orbital fissure, including the oculomotor (CN III), trochlear (CN IV), and abducent (CN VI) nerves, to supply extraocular eye muscles. The supe- rior orbital fissure also contains the superior and inferior oph- thalmic veins and sympathetic fibers from the carotid plexus. With the broken pool cue residing in close proximity to the superior orbital fissure, the ophthalmic division of the trigem- inal nerve is most likely damaged in this question. Choice A (Optic nerve) is incorrect. The optic nerve (CN II) leaves the orbit via the optic canal to relay vision from the retina to the brain. Because only the superior orbital fissure is involved in this patient, the optic nerve is not likely damaged. Choice B (Facial nerve) is incorrect. The facial nerve enters the petrous temporal bone through the internal acoustic meatus, traveling through this foramen with the vestibulocochlear nerve (CN VII). It has three terminal branches (chorda tympani, greater petrosal, and the main branch of the facial nerve), and none of these branches traverse the superior orbital fissure, which was involved in this patient. Choice C (Mandibular division of trigeminal nerve) is incorrect. The mandibular (third) division of the trigeminal nerve (CN V3) supplies general sensation to the skin of the lower lip, chin, cheek, and even the anterior auricular and lateral scalp. This sensory innervation is sup- plied via three cutaneous nerves: mental, buccal, and auricu- lotemporal. The mandibular division of the trigeminal nerve also supplies innervation to the mandibular teeth and gingivae via the inferior alveolar nerve, and it is the only division of CN V that supplies motor innervation. CN V3 enters the middle cranial fossa through the foramen ovale, so it would not be affected by damage in the superior orbital fissure. Choice D (Maxillary division of trigeminal nerve) is incorrect. The maxillary (second) division of the trigeminal nerve (CN V2) supplies sensory (cutaneous) innervation to the skin to the lower eyelid, cheek, upper lip, upper dentition and gingivae, and lateral aspects of the nose. CN V2 enters the middle cra- nial fossa through the foramen rotundum, so it would not be affected by damage in the superior orbital fissure.

While participating in a bar fight, the orbit of a 25-year-old man is pierced by a broken pool cue stick, which extends back to the superior orbital fissure. Which of the following nerves is most likely damaged? (A) Opticnerve (B) Facial nerve (C) Mandibular division of trigeminal (D) Maxillary division of trigeminal (E) Ophthalmic division of trigeminal

E: Parasympathetic fibers via the chorda tym- pani nerve. All the salivary glands receive their secretomotor innervation from the parasympathetic nervous system. Both the submandibular and sublingual salivary glands are sup- plied by the facial nerve by way of its chorda tympani branch.The chorda tympani nerve originates from the distal part of the main trunk of the facial nerve and runs across the middle ear cavity. It earns its name at this location by its thread-like form lying across the deep surface of the tympanic mem- brane and the handle of the malleus. The nerve then enters the infratemporal fossa and merges with the lingual nerve (a branch of the mandibular division of the trigeminal nerve, or CN V3). The parasympathetic fibers synapse in the sub- mandibular ganglion, with the postsynaptic fibers passing to the submandibular and sublingual salivary glands and to mucus glands in the oral floor. Remember, the chorda tympani nerve also carries taste fibers from the anterior two thirds of the tongue. The remaining salivary gland, the parotid, is sup- plied by the glossopharyngeal nerve, by way of the following pathway: tympanic nerve/lesser petrosal nerve/otic ganglion/ auriculotemporal nerve. Choice A (Parasympathetic fibers via the inferior alveolar nerve) is incorrect. The inferior alveolar nerve is a branch of the mandibular division of the trigeminal nerve (CN V3) within the infratemporal fossa. It gives off the mylohyoid nerve, which innervates the mylohyoid and ante- rior digastric muscles, and then enters the mandible traversing the mandibular foramen. Most of its distribution is concerned with general sensation in the lower jaw (including the teeth and gums) and the skin overlying the mandible, including the lower lip. It has no direct role in secretion of any salivary gland. Choice B (Sympathetic fibers via the lingual nerve) is incorrect. Sympathetic neurons do not supply the salivary glands. The lingual nerve does carry autonomic fibers to the submandibular and sublingual salivary glands. However, this autonomic innervation is derived from parasympathetic fibers of the facial nerve/chorda tympani nerve pathway. Choice C (Parasympathetic fibers via the mandibular branch of the facial nerve) is incorrect. The (marginal) mandibular branch of the facial nerve is one of the five terminal branches of the main trunk of the facial nerve innervating the muscles of facial expression. The (marginal) mandibular branch of the facial nerve does not carry autonomic fibers to any salivary glands despite the fact that this nerve transverses the parotid gland. Choice D (Sympathetic fibers via the hypoglossal nerve) is incorrect. Again, the sympathetic division does not supply the salivary glands. The hypoglossal nerve may carry a small hitchhiking sympathetic bundle for part of its course. How- ever, its dominant content is motor fibers to the intrinsic and most of the extrinsic skeletal muscles of the tongue.

While planning a delicious dinner for his former anatomy professors, a doctor finds himself salivating at the thought of the feast. What description accurately describes the secretomotor pathway for innervation of the submandibular gland? (A) Parasympathetic fibers via the inferior alveolar nerve (B) Sympathetic fibers via the lingual nerve (C) Parasympathetic fibers via the mandibular branch of the facial nerve (D) Sympathetic fibers via the hypoglossal nerve (E) Parasympathetic fibers via the chorda tympani nerve

A: Greater petrosal nerve. Secretomotor con- trol of the lacrimal gland is provided by parasympathetic neu- rons derived from the facial nerve. These fibers branch from the facial nerve as the greater petrosal nerve at the geniculum of the facial canal, within the petrous part of the temporal bone. This parasympathetic nerve leaves the temporal bone and courses along the floor of the middle cranial fossa on its way to the foramen lacerum, pterygoid canal, and pterygo- palatine fossa. It is readily damaged in trauma to the floor of the middle cranial fossa. Lesion of the nerve will result in loss of emotional tearing plus reduced mucus secretion in the nasal cavity (dry nasal passages). Choice B (Lesser petrosal nerve) is incorrect. The lesser petrosal nerve is a parasympathetic bundle derived from the tympanic branch of the glossopha- ryngeal nerve in the middle ear. It is the presynaptic (pregan- glionic) element in the secretomotor pathway to the parotid gland via the otic ganglion. Choice C (Deep petrosal nerve) is incorrect. The deep petrosal nerve is a sympathetic bundle that branches off the carotid plexus at the exterior base of the skull. It joins with the greater petrosal nerve to form the nerve of the pterygoid canal that travels through the pterygoid canal into the pterygopalatine fossa. The deep petrosal nerve may be damaged in a fracture of the floor of the middle cranial fossa. However, it does not control lacrimation, and thus is not the element related to the deficit here. Obviously, you must take care to differentiate the three petrosal nerves in accounting for autonomic relations in the head. Choice D (Lacrimal nerve) is incorrect. The lacrimal nerve is a branch of the ophthalmic division of the trigeminal nerve within the orbit. It conveys general sensory fibers from the upper eyelid, conjunctiva, and lacrimal sac. Further, its distal portion carries the postsynap- tic parasympathetic secretomotor fibers to the lacrimal gland. However, the lacrimal nerve itself would not be damaged by a fracture within the middle cranial fossa. Choice E (Chorda tympani nerve) is incorrect. The chorda tympani nerve is a branch of the facial nerve that runs through the middle ear cavity and continues into the infratemporal fossa. It carries parasympathetic secretomotor fibers to two salivary glands, the submandibular and sublingual glands, via the submandib- ular ganglion. It also conveys taste sensation from the anterior part of the tongue.

A 12-year-old boy suffers a fracture of the floor of the right side of the middle cranial fossa during an automobile acci- dent. Subsequent physical examination reveals he is devoid of emotional tearing on the ipsilateral side. Which of the follow- ing nerves is most likely damaged? (A) Greater petrosal nerve (B) Lesser petrosal nerve (C) Deep petrosal nerve (D) Lacrimalnerve (E) Chorda tympani nerve

D The facial vein communicates directly with the inferior ophthalmic vein which communicates directly with the cavernous sinus.

A 35-year-old woman is hospitalized due to cavernous sinus thrombosis resulting from an infection on the face. Which of the following is the most direct route for spread of infection from the face to the cavernous sinus? A Basilar venous plexus B Deep facial vein C Pterygoid venous plexus D Superior ophthalmic vein

B,F B and F are both correct. In oculomotor nerve palsies the lateral rectus muscle is functional and the patient can abduct the affected eye. The superior oblique muscle is also intact, leading to a slight depression of the abducted eye (although its pull is minimized, the superior oblique muscle can still act on the transverse axis when the eye is abducted). When adducting the eye from the abducted position, the eye will rotate along the vertical axis to primary position as a consequence of inhibition of the lateral rectus muscle, but in the absence of contraction of the medial rectus muscle, it cannot rotate beyond that point.

A 45-year-old female presents to her primary care physician with complaints of a droopy right eyelid, double vision, an increased sensitivity to light, and difficulty reading at a close distance. Her physician suspects a cranial nerve injury. In performing an H-Test on this patient, which of the following findings are expected? (More than one answer possible) A The pupil of the affected eye is slightly elevated when abducted B The pupil of the affected eye is slightly depressed when abducted C The pupil of the affected eye is neither elevated nor depressed when abducted D The pupil of the affected eye is slightly elevated when adducted fully E The pupil of the affected eye is slightly depressed when adducted fully F The pupil of the affected eye cannot move past primary position when attempting adduction

B The oculomotor nerve is a "super" motor nerve; it carries both somatic efferent fibers to skeletal muscles in the orbit (levator palpebrae superioris, superior, medial and lateral rectus, and inferior oblique), and visceral efferent (parasympathetic) fibers to smooth muscles in the iris of the eye (ciliary muscle, constrictor pupillae). This patient's constellation of symptoms is consistent with an injury to the right oculomotor nerve. Common causes of oculomotor nerve injury include pressure on or compression of the oculomotor nerve, and inadequate blood flow to the nerve. Of these the former results from such life-threatening, emergent conditions as herniation of portions of the temporal lobes of the brain through the tentorial notch (transtentorial herniation - due to an enlarging tumor or intracranial bleed), or a growing aneurysm in an artery supplying the brain. Inadequate blood flow typically results from diabetes, high blood pressure (hypertension), and other disorders that affect blood vessels.

A 45-year-old female presents to her primary care physician with complaints of a droopy right eyelid, double vision, an increased sensitivity to light, and difficulty reading at a close distance. Her physician suspects a cranial nerve injury.Which cranial nerve is most likely affected in this patient? A Abducens B Oculomotor C Ophthalmic D Optic E Trochlear

C: Third division of trigeminal nerve. The third (mandibular) division of the trigeminal nerve (CN V3) sup- plies general sensation to the skin of the lower lip, chin, cheek, and even the anterior auricle and the lateral scalp. This sensory innervation is supplied via three cutaneous nerves: mental, buccal, and auriculotemporal. The mandibular divi- sion of the trigeminal nerve also supplies innervation to the mandibular teeth and gingivae via the inferior alveolar nerve. This patient is suffering from trigeminal neuralgia (or tic dou- loureux), often seen in patients suffering from demyelinating diseases such as multiple sclerosis, affecting CN V3. Trigeminal neuralgia is characterized by episodes of pain that occur sud- denly, and debilitating pain can often be triggered by stimuli within the distribution area of the nerve affected, which was seen following eating, talking, or brushing her teeth in this patient. Choice A (First division of the trigeminal nerve) is incorrect. The first (ophthalmic) division of the trigeminal nerve (CN V1) supplies sensory (cutaneous) innervation to the skin of the upper eyelid, anterior aspect of the nose, forehead, and ante- rior scalp. The sensory distribution of the ophthalmic division of the trigeminal nerve does not correlate to the areas of the face affected in this patient, so this option can be eliminated. Choice B (Second division of trigeminal nerve) is incorrect. The second (maxillary) division of the trigeminal nerve (CN V2) supplies sensory (cutaneous) innervation to the skin to the lower eyelid, cheek, and upper lip, upper dentition and gingivae, maxillary sinus, and lateral aspect of the nose. The sensory distribution of the maxillary division of the trigeminal nerve does not correlate to the areas of the face affected in this patient, so this option can be eliminated. Choice D (Buccal branch of facial nerve) is incorrect. The buccal branch of the facial nerve (CN VII) is one of five terminal branches of the main trunk of CN VII, which supplies the muscles of facial expression and other muscles derived from mesoderm in the embryonic second pharyngeal arch. This buccal branch is entirely efferent (motor) in its innervation supplying the buc- cinator muscle and muscles of the upper lip. It does not have a sensory component, so this nerve would not be the source of this patient's pain. Choice E (Marginal mandibular branch of the facial nerve) is incorrect. The marginal mandibular branch of the facial nerve (CN VII) is another one of five terminal branches of the main trunk of CN VII. This nerve only has an efferent (motor) component supplying the muscles of lower lip and chin. It does not have a sensory component, so this nerve would not be the source of this patient's pain.

A 47-year-old woman with a history of multiple sclerosis comes to her doctor complaining of sudden bursts (parox- ysms) of pain in her mandible, especially in the lower lip, mandibular teeth and gingivae, and cheek on her right side. This debilitating pain is often triggered by eating, talking, or brushing her teeth and often gets worse as the day progresses. Which nerve is the source of her pain? (A) First division of trigeminal nerve (B) Second division of trigeminal nerve (C) Third division of trigeminal nerve (D) Buccal branch of facial nerve (E) Marginal mandibular branch of facial nerve

E This patient's symptoms are consistent with an injury to the oculomotor nerve. This nerve reaches the orbit by passing through the superior orbital fissure.

A 48-year-old male patient complains of diplopia (double vision). On neurologic examination he is unable to adduct his left eye and lacks a pupillary reflex on the left side. Where is the most likely location of the lesion resulting in these symptoms? A Inferior orbital fissure B Foramen rotundum C Foramen ovale D Optic canal E Superior orbital fissure

C The basilar and occipital dural sinuses in the cranial cavity communicate directly with the internal vertebral venous plexus (Batson's plexus).

A 65-year-old female with a history of ovarian cancer is found to have metastatic disease in her vertebral column and brain. Which of the following is/are most likely responsible to carrying the malignant cells from the vertebral column into the cranial cavity? A Emissary veins B Internal jugular vein C Basilar venous plexus D Cavernous sinus

E The superior orbital fissure is a large bony opening in the posterior wall of the eye orbit. Cranial nerves III, IV, V1, and VI all pass through the superior orbital fissure to enter the eye orbit. CN II passes into the orbit by passing through the optic canal.

All of the following cranial nerves pass through the superior orbital fissure EXCEPT: A CN III B CN V1 C CN IV D CN VI E CN II

A. Horner's syndrome is a result of interruption of the sympathetic supply to the head. In this syndrome, the dilator pupilae is inactive, which results in unopposed action of the constrictor pupillae m.

An 8-year-old male is admitted to the hospital with a drooping right eyelid (ptosis). The initial diagnosis is Horner's syndrome. Which of the following additional signs on the right side would confirm the diagnosis? A. Constricted pupil B. Dry eye C. Exophthalmos D. Pale, blanched face E. Sweaty face

B The cricothyroid membrane passes from the cricoid cartilage to the inferior border of the thyroid cartilage.

An EMT determines that an emergency cricothyrotomy (cutting a hole in the cricrothyroid membrane) is warranted in a patient with airway collapse and severe layngoedema. Which of the following is the most accurate description of the location of the cricothyroid membrane? A Immediately superior to the thyroid cartilage. B Immediately inferior to the thyroid cartilage. C Immediately inferior to the cricoid cartilage. D Just deep to the isthmus of the thyroid gland. E Immediately inferior to the hyoid bone.

A,F A and F are both correct. Unopposed contraction of the inferior oblique muscle will result in a slight elevation of the affected eye when in primary position (A). Weakness of depression of the eye in individuals with trochlear nerve damage results in diplopia when looking down and is most often noticed when walking down the stairs. The patient's tilting of the head is done to rotate the pupil in the unaffected eye and bring it into alignment with the laterally rotated pupil of the affected eye. Straightening of the head will result in rotational changes in the unaffected eye, but not the affected eye (F).

At Thanksgiving dinner you notice for the first time that your grandfather always holds his head at a slight angle when looking directly at you. Suspecting an extra-ocular muscle deficit of some kind, you seize this opportunity to apply your newly-gained knowledge of eye movements and clinical testing to diagnose his condition. In your initial observation of your "patient" you notice that when looking straight ahead your grandfather's head is tilted towards his right shoulder. To rule out some other explanation for your grandfather's head posture (e.g., torticollis), you ask him to straighten his head while you observe his eyes. Which of the following observations do you expect ? (More than one answer possible) A The pupil of the affected eye elevates slightly as he straightens his head B The pupil of the affected eye depresses slightly as he straightens his head C The pupil of the affected eye neither elevates nor depresses as he straightens his head D The pupil of the affected eye rotates medially as he straightens his head E The pupil of the affected eye rotates laterally as he straightens his head F The pupil of the affected eye does not rotate in either direction as he straightens his head

A,B,C A, B and C are all correct. In the H-test, abduction of the eye aligns the superior and inferior rectus muscles so as to maximize their ability to rotate the eye in the transverse axis, while simultaneously minimizing the ability of the superior and inferior oblique muscles to act on this axis (See University of Toronto H-Test Simulator; https://www.bmc.med.utoronto.ca/anatomia/intro.swf). In this individual elevation and depression of the abducted eye will be normal. Adduction of the eye in the H-test aligns the superior and inferior oblique muscles so as to maximize their ability to produce rotation in the transverse axis, while simultaneously minimizing the ability of the superior and inferior rectus muscles to act on this axis. In an individual with a trochlear nerve injury, the adducted eye can be elevated (inferior oblique muscle), not be depressed (superior oblique muscle). Note that in trochlear nerve injuries, elevation of the affected eye is maximized when the eye is adducted.

At Thanksgiving dinner you notice for the first time that your grandfather always holds his head at a slight angle when looking directly at you. Suspecting an extra-ocular muscle deficit of some kind, you seize this opportunity to apply your newly-gained knowledge of eye movements and clinical testing to diagnose his condition. In your initial observation of your "patient" you notice that when looking straight ahead your grandfather's head is tilted towards his right shoulder. You perform an H-test on your grandfather. Which of the following findings are expected? (More than one answer possible) A The pupil of the affected eye elevates as expected when abducted B The pupil of the affected eye depresses as expected when abducted C The pupil of the affected eye elevates as expected when adducted D The pupil of the affected eye depresses as expected when adducted

B Premature fusion of the coronal and lambdoidal sutures will be compensated for by increased growth at the sagittal and metopic sutures. The resulting skull shape will be wide and tall.

Craniosynostosis of the coronal and lambdoid sutures will result in which of the following skull shapes? A Round and short B Wide and tall C Tall and narrow D Long and narrow

D Premature fusion of the sagittal suture will be compensated for by increased growth at the coronal and lambdoid sutures. The resulting skull shape will be long and narrow.

Craniosynostosis of the sagittal suture will result in which of the following skull shapes? A Round and tall B Round and short C Tall and narrow D Long and narrow

E By virtue of its proximal attachment anterior to the mandibular condyle, the lateral pterygoid muscle (superior head) is the muscle most capable of protruding the mandible.

Following recovery of a surgical repair of a broken mandibular body the strength and symmetry of the muscles of mastication are tested. Which of the following muscles is capable of depressing the mandible? A Masseter B Anterior temporalis C Posterior temporalis D Medial pterygoid E Lateral pterygoid

A. the buccinator muscle forms the skeleton of the check; its contraction during chewing prevents the bolus from slipping laterally into the oral vestibule

Individuals suffering from Bell's palsy often find that food collects in the vestibule of the oral cavity when chewing. Which of the following muscles functions to keep food between the upper and lower teeth during chewing? A. Buccinator B. Lateral pterygoid C. Medial pterygoid D. Masseter E. Orbicularis oris

C unilateral contraction of the SCM simultaneously flexes the neck and rotates the chin in the opposite direction. A congenital shortening of this muscle results in the same posture

On examination of a newborn baby it is noted that her head posture is abnormally tilted toward the right and her chin is elevated and turned toward the left side. This posture is consistent with congenital shortening of which of the following muscles? A. Anterior scalene B. Omohyoid C. Sternocleidomastoid D. Trapezius

B,D,F,G CN III, VII, IX, X CNs III, VII, IX, and X all emerge from the brainstem with preganglionic parasympathetic nerve fibers bundled within. Although most of these nerve fibers remain within their cranial nerve of origin, not all do. Some preganglionic nerve fibers will leave their cranial nerve of origin and join other cranial nerves (usually branches of the trigeminal nerve) to reach their effector organs in the periphery.

Which of the following cranial nerves have a parasympathetic visceral motor/efferent component? (select all that apply) A CN II B CN III C CN V D CN VII E CN VIII F CN IX G CN X H CN XI I CN XII

E The loose tissue of the loose areolar layer of the scalp is the "danger zone" where infections may develop.

Which of the following layers of the scalp is regarded as the "danger zone"? A Skin B Occipitofrontalis & epicranial aponeurosis C Periosteum D Dense connective tissue E Loose areolar tissue

B Buccinator is a muscle of facial expression and is innervated by the facial nerve. The rest are muscles of mastication and are innervated by CNV3.

Which of the following muscles is NOT innervated by the trigeminal nerve? A medial pterygoid B buccinator C temporalis D masseter E lateral pterygoid

B,D,E B-glossopharyngeal- posterior 1/3 D- facial - anterior 2/3 E- vagus - root of tongue The facial nerve (CN VII) carries taste from the anterior 2/3 on the tongue, and glossopharyngeal nerve (CN IX) carries taste from the posterior 1/3 of the tongue, the vagus nerve (CN X) carries taste from the "root" of the tongue in the oropharynx.

Which of the following nerves carry taste sensation? (select all that apply) A Trigeminal B Glossopharyngeal C Hypoglossal D Facial E Vagus

B,C, E

Which of the following nerves have both sensory and motor components? (select all that apply) A CN II B CN V C CN VII D CN VIII E CN X F CN XI

C Look up and away from injured side Unilateral contraction of the sternocleidomastoid muscle rotates the chin in the opposite direction.

Which of the following statements best describes the physical presentation of an upright patient with torticollis due to excessive contraction of the right sternocleidomastoid muscle? A Head flexed forward in the midline B Neck flexed and chin rotated up and to the right C Neck flexed and chin rotated up and to the left D Head extended in the midline

B,D,F,H, J

Which of the following venous structures communicates directly with the cavernous sinus? (select all that apply) A Facial vein B Superior ophthalmic vein C Superior sagittal sinus D Inferior ophthalmic vein E Pterygoid plexus of veins F Inferior petrosal sinus G Deep facial vein H Superior petrosal sinus I Basilar venous sinus J Intercavernous sinus

D Recall that the cell bodies of preganglionic sympathetic neurons are all located in the T1-L2/L3 regions of the spinal cord. Those preganglionic sympathetic neurons destined to innervate head & neck structures are located in the T1-T4 regions of the spinal cord.

Your patient has a lesion in the region of the CNS that contains the cell bodies of the preganglionic sympathetic neurons that innervate structures in the head. The lesion is located in which of the following locations? A Brainstem B C1-C4 regions of spinal cord C C5-C8 regions of spinal cord D T1-T4 regions of spinal cord E T5-T8 regions of spinal cord


Related study sets

Earth Science Exam 5- Ground Water

View Set

English 11 Final Exam Literature Review

View Set

policy provisions and contract law

View Set

CNA 122 | Ch. 6 & Ch. 7, Installing Hyper-V

View Set

Practice for Test 2 (Chapters 9,10,11, & 16)

View Set

Section 5 Intro to buisness test

View Set